Cosmetic Abdominoplasty/Liposuction/Abdominoplasty Flashcards

1
Q

For women undergoing abdominoplasty after massive weight loss, which of the following is the best position of the navel? A) Along the line drawn between the iliac crests B) At the horizontal level of the tenth ribs C) Between the first and second tendinous inscriptions D) In the midline 10 cm above the vulvar commissure E) One-third of the distance from the xiphoid to the pubis

A

The correct response is Option A. As a result of the rapid increase in the number of bariatric surgical procedures performed each year, the frequency of body contouring procedures has risen concomitantly over the past decade. Among the most popular of these is abdominoplasty for resection of redundant skin on the torso. When designing the outline of skin to be resected, among the preeminent concerns is maximizing the aesthetic result of the procedure. Removal of the redundant panniculus may involve a low transverse incision only or a more extensive resection such as the fleur-de-lis or corset pattern incisions. The length of the navel stalk may limit any transposition of this structure. Accordingly, one must plan for sitting the navel in an aesthetically pleasing location to complement the finished result. The umbilicus is typically inset along a horizontal line that spans the iliac crests. This will result in the most natural appearance for most individuals. A location 10 cm above the anterior vulvar commissure would result in a placement that is unnaturally low. The other options would yield a position of the umbilicus that is too high.

How well did you know this?
1
Not at all
2
3
4
5
Perfectly
2
Q

A 51-year-old woman comes to the office for consultation for abdominal and lower extremity liposuction. The procedure is expected to last approximately 2.5 hours. BMI is 30 kg/m2. The patient takes an oral contraceptive. The patient reports having had a small venous thromboembolism (VTE) during lumpectomy for breast cancer that took place in her early 40s. Which of the following factors increases the risk of VTE and Caprini risk assessment score most significantly? A) Age B) History of malignancy C) History of VTE D) Length of surgery E) Use of an oral contraceptive

A

The correct response is Option C. In this patient, the highest Caprini risk factor is the history of a VTE, which carries a score of 3. The length of surgery is greater than 45 minutes so it would be considered major and would carry a score of 2, as would her history of malignancy. Age, obesity, and contraceptive use all carry a score of 1. This gives the patient a Caprini score of 10.

How well did you know this?
1
Not at all
2
3
4
5
Perfectly
3
Q

Which of the following symptoms meet the current Medicare guidelines for approval of abdominal lipectomy/panniculectomy following massive weight loss? A) Neck and back pain B) Psychological distress C) Unsatisfactory appearance D) Diastasis recti E) Recurrent intertrigo

A

The correct response is Option E. Medically necessary criteria for Medicare approval of abdominal lipectomy/panniculectomy include: Inability to walk normally Chronic pain and ulceration created by the abdominal skin fold When the panniculus hangs below the level of the pubis Intertrigo of the pannus that is persistent or recurrent over a 3-month period while receiving appropriate medical therapy Stable weight for at least 6 months and 18 months after gastric bypass surgery According to Medicare guidelines, the other four options listed are not considered reasons that make this procedure medically necessary: Treatment of neck and back pain Repairing abdominal wall laxity or diastasis recti Improving appearance Treating psychological symptomatology

How well did you know this?
1
Not at all
2
3
4
5
Perfectly
4
Q

A 48-year-old woman undergoes liposuction of the abdomen, flanks, and thighs. Liposuction is performed using 4 L of infiltration fluid. Each liter is mixed with 50 mL of 2% plain lidocaine and 1 mL of 1:1000 epinephrine. At which of the following times after infiltration are concentrations of lidocaine in the blood expected to be the highest in this patient? A) Immediately after injection B) 1 hour after surgery C) 2 to 4 hours after surgery D) 8 to 18 hours after surgery E) 24 to 48 hours after surgery

A

The correct response is Option D. The safe dosage of lidocaine in liposuction is 35 to 55 mg/kg. Peak lidocaine levels are reported to be 8 to 18 hours after infiltration. Oftentimes, patients are discharged to home when peak levels occur. Surgeons should keep this in mind when calculating lidocaine dosage.

How well did you know this?
1
Not at all
2
3
4
5
Perfectly
5
Q

A 35-year-old woman comes to the office for consultation regarding a tummy tuck. She wants the scar as low as possible, but she does not want a lower vertical midline scar. Physical examination shows mild upper and lower abdominal skin excess and rectus abdominis diastasis. BMI is 27 kg/m2. Abdominoplasty and repair of diastasis are planned. Intraoperatively, there is marked tension on the lower central abdominal flap closure. Which of the following is the most appropriate maneuver to decrease the tension on the repair? A) Creation of a 3-cm transverse umbilical opening B) Liposuction of the upper abdomen C) Relaxing incision of the external oblique fascia D) Scoring of Scarpa fascia to the dermis E) Use of progressive tension sutures

A

The correct response is Option E. Progressive tension sutures are placed from Scarpa’s fascia to the abdominal wall fascia. This helps close the dead space, minimize flap movement, and minimize seroma rate. When placing these sutures with progressive tension, final tension on the abdominal suture line can be lessened. In so doing, healing complications can be reduced. The more common method for reducing tension on flap closure is to close the native umbilical skin opening in a vertical direction. This technique leaves a vertical incision in the midline of the abdominal flap. The need for revision of this scar is not infrequent. Further, most patients want to avoid this scar. Creating a 3-cm transverse incision for the umbilicus would decrease the tension on the flap; however, the appearance of the umbilicus would be aesthetically unacceptable. Relaxing incision of the external oblique fascia is used for closure of ventral herniorrhaphy and would not lessen skin flap tension. Scoring Scarpa’s fascia to the dermis would injure the subdermal vascular plexus, on which the vascularity of the abdominal flap depends. Upper abdominal liposuction can be performed at the same time as abdominoplasty, provided care is taken to maximize preservation of the lateral rectus perforators. It would not be a solution to minimize skin tension.

How well did you know this?
1
Not at all
2
3
4
5
Perfectly
6
Q

Compared with liposuction, which of the following is the greatest advantage of cryolipolysis? A) No procedural discomfort B) No risk of bruising C) No risk of posttreatment swelling D) No surgical intervention E) Shorter duration of treatment time

A

The correct response is Option D. Nonsurgical fat freezing treatment (CoolSculpting) is a method of noninvasive fat reduction that occurs via the use of cryolipolysis. It is a noninvasive technique that takes place in an office setting and does not involve the use of surgery. Following CoolSculpting treatment, patients have some aspect of bruising and swelling, which can last for up to several weeks. Results of CoolSculpting treatments typically take 3 to 4 months to develop. Results are additive, however, with multiple treatments. CoolSculpting involves the utilization of different applicators that have been developed to treat various parts of the body effectively. Each actual treatment is for 1 hour; however, most areas of the body may require multiple treatments to be effectively managed. Treatment times for an abdomen can range from 2 to 6 hours alone, whereas outer thighs are treated for at least 2 hours per side and inner thighs at least 1 hour per side. Although not all of these treatments need to be performed in one sitting, the total time for CoolSculpting treatments for most areas of the body are much longer than it would take to surgically perform liposuction. In general, however, CoolSculpting is a less expensive treatment option compared with liposuction as there is no fee necessary for anesthesia or operating-room costs.

How well did you know this?
1
Not at all
2
3
4
5
Perfectly
7
Q

During routine brachioplasty, which of the following nerves is/are most likely at risk during typical dissection? A) Lateral antebrachial cutaneous nerve B) Medial antebrachial cutaneous nerve C) Sensory branches of the axillary nerve D) Sensory branches of the radial nerve

A

The correct response is Option B. The medial antebrachial nerve is most at risk for injury during routine brachioplasty surgery secondary to its superficial location within the subcutaneous tissue within the area of typical skin and soft-tissue excision. This nerve arises from the medial cord of the brachial plexus 78% of the time and from the lower trunk in 22%. After emerging from the axilla, the medial antebrachial cutaneous nerve travels medial to the brachial artery and lies adjacent to the basilic vein at the distal upper arm. In the distal or mid brachium, this nerve pierces the deep fascia to become very superficial running above the deep fascia at an average of 14 cm proximal to the medial epicondyle. Despite some minor anatomical variability, this nerve has been found to be consistently present in the deep plane of dissection for the standard brachioplasty technique.

How well did you know this?
1
Not at all
2
3
4
5
Perfectly
8
Q

A 34-year-old woman is evaluated for body contouring after Roux-en-y gastric bypass surgery 6 months ago. There is no evidence of malabsorption. BMI is 36.3 kg/m2. She had a 75-lb (34-kg) weight loss and is actively losing weight. The patient reports low back pain. Which of the following is the most appropriate next step in management? A) Liposuction B) Panniculectomy C) Revision of the gastric bypass surgery D) Upper GI series E) Observation

A

The correct response is Option E. After bariatric surgery, patients can continue to lose weight as a result of the surgical procedure for approximately 2 years. Thus, most recommendations call for waiting until patients are 12 to 18 months out from their bariatric surgery and at a stable weight for 3 to 6 months. Ideally, patients should be within 10 to 15% of their goal weight. In this case, the patient is still within the time frame of active weight loss, and notes that she is actively losing weight. Thus, the appropriate answer is to wait until weight loss has stabilized. Because this patient is actively losing weight and there are no clinical findings of any issues such as malabsorption, there is no indication currently to evaluate her with an upper GI series or revise her bypass. In addition, as noted above, the risks for surgery are increased at this patient’s BMI. Thus, elective liposuction or panniculectomy is not appropriate at this time. Furthermore, because the patient is actively losing weight, the risk for revision surgery to address additional skin laxity that may develop with further weight loss makes undertaking these procedures not appropriate at this point in time.

How well did you know this?
1
Not at all
2
3
4
5
Perfectly
9
Q

Compared with standard suction-assisted lipectomy, laser-assisted liposuction has been shown to decrease which of the following? A) Contour irregularities B) Ecchymosis C) Postoperative pain D) Skin necrosis E) Swelling

A

The correct response is Option C. A prospective, randomized, double-blind study involving human subjects compared the effects of laser-assisted lipoplasty with suction-assisted lipoplasty. No significant difference was noted between the two groups with respect to cosmetic outcome, ecchymosis, edema, skin retraction, or surgical time. The only measured potential benefit of the laser-assisted technique was an overall decrease in postoperative pain.

How well did you know this?
1
Not at all
2
3
4
5
Perfectly
10
Q

A 35-year-old woman comes to the office for lipodystrophy of the upper arms. Physical examination shows negligible skin laxity; pinch test shows a thickness of 3 cm of the entire upper arm and the chest wall. Which of the following is the most appropriate surgical intervention? A) Extended brachioplasty B) Limited medial brachioplasty C) Mini brachioplasty D) Suction-assisted lipectomy E) Traditional brachioplasty

A

The correct response is Option D. Skin laxity is the single greatest determinant of whether liposuction is an appropriate modality in an algorithmic approach to upper arm lipodystrophy. The determination of excessive fat can be made by the pinch test, and patients with greater than 1.5 cm of fat on a pinch test may be candidates. The classification of lipodystrophy, described by Rohrich et al., includes skin excess, fat excess, and the location of skin excess. Where there is skin excess, the skin must be excised for a favorable result. Liposuction alone can exacerbate the appearance and presence of excess skin. Similarly, skin laxity is a predictor of liposuction success. With marked laxity, the skin is unlikely to have enough elastic properties to retract. Although there are some papers that show increased retraction of skin with laser liposuction, this has not been shown to be a consistent result in large-scale studies.

How well did you know this?
1
Not at all
2
3
4
5
Perfectly
11
Q

A 37-year-old woman reports nontender swelling of the lumbar area after undergoing lower body lift following massive weight loss. Examination shows tense swelling and a positive fluid wave test. Percutaneous needle aspiration is performed on a weekly basis, and fluid is still present after three aspirations of 150 mL each of a clear, yellowish serum. Which of the following is the most appropriate next step in management? A) Compression B) Operative incision and drainage C) Placement of a closed suction drain tube D) Use of an ipsilateral gluteus maximus muscle advancement flap

A

The correct response is Option C. Among the most common complications following body contouring for post-massive weight loss-induced skin laxity is seroma, occurring in up to 35 to 50% of patients. To minimize the risk for seroma, preoperative nutritional repletion, especially for protein, and intraoperative use of closed suction drains, aggressive minimization of dead space, limited degree of skin flap undermining, and use of well-fitted elastic compression garments are among the techniques that are commonly recommended. None of these, even in combination, can completely guarantee the elimination of this complication. After several ineffective aspirations, compression garments alone are not likely to eliminate a seroma for this patient. The most appropriate next management measure for this patient’s seroma is ultrasound-directed percutaneous closed suction drain placement. Operative incision and drainage is not indicated unless the closed drain fails, following failed sclerosant therapy, or if the seroma is shown to be infected. A muscle flap is not indicated in this setting.

How well did you know this?
1
Not at all
2
3
4
5
Perfectly
12
Q

A 35-year-old woman with a history of hypertension is evaluated for body contouring. Physical examination shows generalized abdominal adiposity and moderate infraumbilical pannus. The patient undergoes abdominoplasty and large-volume liposuction after induction of epidural anesthesia. A super-wet technique is used and a total volume of 5500 mL is removed. Which of the following factors poses the greatest risk of death for this patient? A) Abdominoplasty B) Epidural anesthesia C) Hypertension D) Liposuction volume E) Super-wet technique

A

The correct response is Option A. The cumulative effect of multiple procedures performed during a single operation increases the potential that complications may develop. Large-volume liposuction, combined with other procedures such as abdominoplasty, can cause serious complications. Death associated with isolated lipoplasty is rare (0.0021%, or one per 47,415), but mortality increases significantly when lipoplasty is combined with other procedures. When combined with non-abdominoplasty procedures, lipoplasty mortality increases to one per 7314; when combined with abdominoplasty, with or without other procedures, the lipoplasty mortality increases to one per 3281. The presumed benefits of combined procedures must thus be weighed against potential untoward events. Studies indicate that epidural anesthesia combined with the infusion of anesthetic infiltrate provides patients with a consistent intraoperative comfort level. Data from the few anesthesia studies that have specifically assessed patients undergoing liposuction confirm the safety of general anesthesia, epidural anesthesia, spinal anesthesia, moderate sedation, and local anesthesia for this procedure. It should be noted, however, that epidural anesthesia and spinal anesthesia can cause vasodilation and hypotension, thereby necessitating the administration of excess fluid and increasing the risk of fluid overload. Based on the patient’s history, physical examination, review of systems, laboratory testing, and/or a medical specialist’s evaluation, the physician should select the patient’s American Society of Anesthesiologists (ASA) physical classification rating: Type 1: A normal healthy patient; Type 2: A patient with mild systemic disease; Type 3: A patient with severe systemic disease; Type 4: A patient with severe systemic disease that is a constant threat to life. ASA Type 1 and Type 2 patients are candidates for ambulatory and office-based surgery. The patient described is a Type 2 patient, a classification rating that represents patients who have any of the following conditions that are under control without systemic compromise: diabetes mellitus, hypertension, asthma, gastroesophageal reflux disease, peptic ulcer disease, hematologic disorders, arthritis, and neuropathy. Large-volume liposuction is defined as the removal of 5000 mL or greater of total aspirate during a single procedure. A review of the scientific literature shows that there are no scientific data available to support a specific volume maximum at which point liposuction is no longer safe. The super-wet technique, introduced in the mid-1980s, uses larger volumes of subcutaneous infiltrate, whereby 1 to 2 mL of solution is infused for each 1 mL of fat to be removed. The infiltrate solution consists of saline or Ringer’s lactate with epinephrine and, in some cases, lidocaine. Using this method, blood loss generally decreases to less than 1 to 2% of the aspirate volume.

How well did you know this?
1
Not at all
2
3
4
5
Perfectly
13
Q

An otherwise healthy 41-year-old woman who underwent Roux-en-Y gastric bypass surgery 24 months ago, followed by a 120-lb (54-kg) weight loss that she maintained for 6 months, undergoes plastic surgery evaluation for a panniculectomy. Medical history includes hypothyroidism that is controlled with levothyroxine. Preoperative cardiovascular examination shows no abnormalities, and results of a pregnancy test on the day of surgery are negative. She undergoes panniculectomy and thigh lift, and on extubation, the patient is lethargic and confused. ECG shows sinus tachycardia, and she remains somnolent and confused. Analysis of thyroid-stimulating hormone and cardiac enzymes, chest x-ray study, and ventilation-perfusion scan show no abnormalities. Which of the following is the most likely diagnosis? A) Acute thyroiditis B) Diabetic ketosis C) Pulmonary embolism D) Undiagnosed pregnancy E) Vitamin B1 (thiamine) deficiency

A

The correct response is Option E. Thiamine deficiency is most often identified shortly after bariatric surgery but can be diagnosed later. Some patients can develop Wernicke-Korsakoff encephalopathy (WKE). Body stores of thiamine can last from 3 to 6 weeks, and thiamine deficiency is more associated with decreased dietary intake. Although clinical manifestations are very uncommon, and WKE is considered a rare complication, approximately 11% of patients who have undergone Roux-en-Y gastric bypass surgery and take vitamin supplementation show evidence of thiamine deficiency 2 years postoperatively. The hallmark of thiamine deficiency is neurologic symptoms, but in contrast to WKE, patients rarely exhibit confusion, ataxia, and oculomotor abnormalities. If thiamine deficiency is not recognized and treated, it can have devastating results, including irreversible brain damage and death. Full nutritional workup of patients is critical. Confusion is a symptom of diabetic ketosis, but diabetic ketosis is notable for signs of dehydration and excessive thirst or urination, and is associated with Kussmaul respirations. Acute thyroiditis has symptoms of pain and swelling of the anterior neck. Pulmonary embolism can have symptoms of respiratory distress and right-sided heart strain. Early pregnancy often results in nausea, but not lethargy and confusion.

How well did you know this?
1
Not at all
2
3
4
5
Perfectly
14
Q

A 39-year-old woman is evaluated because she is dissatisfied with the appearance of her abdomen. She has had five pregnancies with two full-term deliveries and three second-trimester miscarriages. She takes no birth control pills. Physical examination shows wide diastasis with excess abdominal skin. BMI is 28 kg/m2. Abdominoplasty with translocation of the umbilicus is planned. This patient is at increased risk for which of the following complications? A) Deep vein thrombosis B) Hematoma C) Infection D) Recurrent diastasis E) Wound dehiscence

A

The correct response is Option A. A history of two late-term miscarriages stands out as a significant risk factor for thrombophilia (inherited and acquired) and subsequent deep vein thrombosis. It is imperative that these patients be identified and further evaluated. If abdominoplasty is performed, chemoprophylaxis is required. The most common inherited thrombophilia is factor V Leiden, which is present in 3 to 7% of the Caucasian population. Multiple inherited thrombophilic conditions can be present in the same individual. Studies have shown that complication rates in abdominoplasty increase in patients with a BMI greater than or equal to 30 kg/m2. Based upon history and physical examination alone, the other complications of hematoma, infection, recurrent diastasis, or wound dehiscence should not be significantly increased.

How well did you know this?
1
Not at all
2
3
4
5
Perfectly
15
Q

A 32-year-old woman is evaluated for lipodystrophy of the central abdomen with skin laxity. She desires volume reduction as well as improvement of the skin laxity. The patient is scheduled for superficial liposuction of the abdomen. Which of the following postoperative complications is most likely in this patient? A) Contour irregularities B) Hyperpigmentation C) Infection D) Seroma E) Skin necrosis

A

The correct response is Option A. The subcutaneous fat of the abdomen is anatomically arranged in two layers: superficial and deep. The superficial adipose layer is located 1 to 2 mm below the dermis and is dense and compact with numerous septations. The deep adipose layer is loose and areolar with few septa. Conventional or traditional liposuction is performed within the deep adipose layer with larger cannulas. Superficial liposuction or subdermal liposuction involves the removal of fat from the superficial compartment found 1 to 2 mm below the dermis, disrupting the extensive septations. Most complications following liposuction are minor and resolve without further surgical intervention. However, the most common complications following superficial liposuction are contour irregularities. Less common complications associated with superficial liposuction include seroma, hyperpigmentation, infection, hypertrophic scar, chronic induration, skin necrosis, and infection.

How well did you know this?
1
Not at all
2
3
4
5
Perfectly
16
Q

A 40-year-old woman, gravida 2, para 2, with abdominal laxity and rectus diastasis is scheduled to undergo abdominoplasty with rectus plication. Which of the following intraoperative nerve blocks is likely to provide postoperative analgesia to the greatest area of lower abdominal skin for this patient? A) Direct midline injection of plication area B) Iliohypogastric nerve block C) Ilioinguinal nerve block D) Subcostal nerve block E) Transversus abdominis plane block

A

The correct response is Option E. Innervation of the anterolateral abdominal wall arises from the anterior rami of spinal nerves T7 to L1. These branches include the intercostal nerves (T7-T11), the subcostal nerve (T12), and the iliohypogastric and ilioinguinal nerves (L1). Because these nerves travel in the plane between the transversus abdominis and internal oblique muscles, they can be conveniently blocked in this area with a single transversus abdominis plane (TAP) block on each side. Although the other nerve block techniques are frequently used in combination, each covers a smaller territory or single dermatome. The TAP block may be performed via several different approaches. Most experts agree that there is a reliable block of the T10 to L1 dermatomes when the lateral approach from the triangle of Petit is used. The subcostal approach of the TAP block can give a more cephalad block. The combination of bilateral TAP blocks and rectus sheath injections has been found to decrease the need for postoperative narcotic use after abdominoplasty. It has also been useful for patients receiving transverse rectus abdominis musculocutaneous (TRAM) and deep inferior epigastric artery perforator (DIEP) flaps. The TAP block was also found to be superior to conventional ilioinguinal and iliohypogastric nerve blocks in a comparison study of open inguinal hernia repairs.

How well did you know this?
1
Not at all
2
3
4
5
Perfectly
17
Q

Which of the following is the most common reason for reoperation after brachioplasty in patients who have undergone significant weight loss? A) Hematoma B) Hypertrophic scar C) Infection D) Seroma E) Wound dehiscence

A

The correct response is Option B. In general, patients are satisfied with the results of brachioplasty. Complications are not uncommon and some studies report complication rates as high as 50%. Most of these complications are minor in nature. Wound dehiscence is managed with dressing changes. Hematoma and seromas are typically managed with observation or aspiration. In the case of infections, the majority are managed with oral antibiotics. The most common reason for additional surgery after brachioplasty is to revise the scar. Many patients will have an unfavorable scar that either hypertrophies or widens. This can be addressed by a scar revision, laser treatment, or triamcinolone acetonide (Kenalog) injection.

How well did you know this?
1
Not at all
2
3
4
5
Perfectly
18
Q

A 66-year-old woman undergoes minimal-incision brachioplasty to treat bilateral upper extremity skin laxity. Height is 5 ft 6 in (167.6 cm) and weight is 140 lb (63.5 kg). BMI is 22.6 kg/m2. Which of the following suturing techniques is most appropriate to minimize widening of the scar? A) Arm dermis to axillary dermis B) Arm dermis to axillary dermis to axillary fascia C) Arm dermis to axillary dermis to pectoralis major fascia D) Axillary dermis to lateral pectoralis major tendon E) Axillary dermis to superficial pectoralis minor fascia

A

The correct response is Option B. The popularity of brachioplasty has significantly increased in America over the past decade, in large part because of the number of patients undergoing bariatric surgery for morbid obesity. In this population of patients who have undergone massive weight loss, the severity of excess upper extremity skin mandates a long, often hypertrophic scar in the bicipital groove, which is generally accepted by patients. By contrast, older patients with skin laxity but little lipodystrophy and no history of significant weight change are hard-pressed to accept this visible and often unpredictable scar. For this population, minimal-incision brachioplasty has emerged as an excellent solution and has itself been increasing in popularity over the past few years. In minimal-incision brachioplasty, incisions are limited to the axilla. The procedure is usually combined with suction lipoplasty to remove some excess upper extremity fat, to treat dog ears at the proximal and distal extent of the scar, or to facilitate undermining. Several key maneuvers improve the appearance of the scar and are tantamount to achieving high patient satisfaction after this procedure. The benefit of anchoring the superficial fascial system was first recognized by surgeons performing traditional brachioplasty procedures. A similar concept applies to minimal-incision brachioplasty, where anchoring of the arm and axillary dermis to the superficial fascia is seen to have several advantages. As part of a layered closure, this technique distributes tension in a more even and controlled manner, releasing the high tension on the final skin closure and decreasing the risk of a widened scar. Moreover, these sutures close the dead space within the axilla and recreate the axillary hollow. Suturing the arm dermis to the axillary dermis without fascial reinforcement results in a high-tension closure and often a widened, hypertrophic scar. Suturing the skin to the deeper fascial layers would be difficult and deforming.

How well did you know this?
1
Not at all
2
3
4
5
Perfectly
19
Q

A 35-year-old woman undergoes abdominoplasty and inner thigh liposuction. After the procedure, burning pain radiating down the right anterior thigh is noted. Pain increases when the patient stands and walks. Injury to which of the following nerves is most likely in this patient? A) Genitofemoral B) Iliohypogastric C) Ilioinguinal D) Lateral femoral cutaneous E) Saphenous

A

The correct response is Option D. In several studies of complications of abdominoplasty, the most common nerve injury was to the lateral femoral cutaneous nerve. Symptoms include anterior and lateral thigh burning, tingling, and/or numbness that increase with standing, walking, or hip extension. The genitofemoral nerve supplies the proximal portion of the thigh about the femoral triangle just lateral to the skin that is innervated by the ilioinguinal nerve. Nerve injury may result from hernia repair, but injury to this nerve is rare. The ilioinguinal nerve arises from the fusion of T12 and L1 nerve roots and pierces the transversus abdominis and internal oblique muscles. The nerve then supplies sensory branches to supply the pubic symphysis, the superior and medial aspect of the femoral triangle, and either the root of the penis and anterior scrotum in the male or the mons pubis and labia majora in the female. The nerve can be injured in abdominoplasty and other lower abdominal incisions. Symptoms include paresthesia of the skin along the inguinal ligament. The sensation may radiate to the lower abdomen. Pain may be localized to the medial groin, the labia majora or scrotum, and the inner thigh. The iliohypogastric nerve arises primarily from L1. The distribution of the cutaneous sensation of the iliohypogastric nerve most commonly is a small region just superior to the pubis. The iliohypogastric nerve is rarely injured in isolation. Symptoms include burning pain into the inguinal and suprapubic region. Saphenous nerve symptoms of entrapment may include a deep aching sensation in the thigh, knee pain, and paresthesia in the cutaneous distribution of the nerve in the leg and foot.

How well did you know this?
1
Not at all
2
3
4
5
Perfectly
20
Q

A 39-year-old woman is referred for abdominoplasty. She has a history of severe uterine fibroids, and her gynecologist is planning a total abdominal hysterectomy (TAH). The patient would like to have the abdominoplasty and the TAH performed at the same time. Which of the following is the most appropriate response to this patient’s inquiry? A) Combining the surgeries can be done safely via any approach for TAH B) The gynecologist may perform TAH, but the abdominoplasty flap must be raised first C) The patient’s risk of a thromboembolic event is decreased by having one large surgery D) TAH cannot be combined with abdominoplasty

A

The correct response is Option A. Performing abdominoplasty in combination with other procedures has become a much more common request from patients. Advantages to this approach would include one recovery period versus multiple (which would minimize time away from work and or family, financial advantage to the patient, and a reduced need for multiple hospitalizations and exposure to anesthesia). Several studies have proven the safety of performing abdominoplasty combined with intra-abdominal procedures such as total abdominal hysterectomy (TAH). While the safety profile of combining these surgeries has been well proven, the risk of thromboembolic events is increased due to the extended time of surgery, so that aggressive deep venous thrombus prophylaxis must be administered perioperatively such as set forth by the Plastic Surgery Task Force on Deep Venous Thrombosis Prophylaxis. A TAH may be done via open or laparoscopic approach, depending on the gynecologist’s preference or patient’s chosen method. In addition, as robotically assisted gynecologic procedures have gained widespread acceptance, this approach for a TAH may also be combined with abdominoplasty surgery. Typically, if a laparoscopic or robotic method were chosen, the gynecologist would start the procedure; this way the port scars may be planned such that they can be excised within the abdominoplasty flap to be removed. Although the abdominoplasty flap may be raised off of the fascia before port placement, this usually causes the ports to be more unstable without the added support of the skin and soft tissues to properly hold them in place.

How well did you know this?
1
Not at all
2
3
4
5
Perfectly
21
Q

A 37-year-old man is scheduled to undergo bilateral brachioplasty to correct redundant arm skin due to massive weight loss following gastric bypass surgery. Which of the following methods is most appropriate to minimize sensory cutaneous nerve injury during this procedure? A) Design the longitudinal incision anterior to the bicipital groove B) Divide the intermuscular septum while resecting skin and fat C) Elevate and transpose the basilic vein before closure of the brachioplasty defect D) Leave at least 1 cm of fat over the brachial fascia in the proximal arm E) Maintain a sharp dissection plane deep to the brachial fascia

A

The correct response is Option D. Among the complications that have been reported following brachioplasty are injuries to the sensory nerves that traverse the medial arm. The most likely nerve to be injured is the medial antebrachial cutaneous nerve. It perforates the antebrachial fascia at a fairly constant distance of 14 cm proximal to the medial epicondyle, putting it at a greater risk of injury than either the ulnar or median nerves, both of which lie deep to the brachial fascia. It has a variable relationship with the basilic vein, but this structure is never transposed during brachioplasty. There is a greater danger to nerve injury when the dissection is conducted deeper than the Scarpa’s fascia, therefore dissection deep to the brachial fascia or harvest of the intermuscular septum is contraindicated. A layer of subcutaneous fat at least 1 cm thick should be left over the deep fascia to minimize injury to the sensory nerves in the arm. Placing the longitudinal incision slightly posterior to the bicipital groove decreases chances for injury to the nerve as well.

How well did you know this?
1
Not at all
2
3
4
5
Perfectly
22
Q

A 56-year-old woman comes to the office after gastric bypass surgery with a weight loss of 155 lb (70 kg). Weight is 143 lb (65 kg) and BMI is 24 kg/m2. She desires an improved appearance of the upper arms. A photograph is shown. Which of the following is the most appropriate surgical correction of this deformity? A) Liposuction alone B) Liposuction followed by brachioplasty C) Limited medial brachioplasty D) Full brachioplasty

A

The correct response is Option D. The most appropriate management for this condition is brachioplasty. Surgical management of the upper arm, particularly after massive weight loss, is dependent upon the ratio of fat and skin laxity. There are several classifications, but all address this ratio. Those with a great deal of skin laxity and little fat are best treated by direct excision (brachioplasty). Those patients who have little skin laxity (and good skin tone) and marked fat may benefit from liposuction alone. Those patients in the middle, with skin laxity and residual upper arm fat, are likely to benefit from a combination approach, either in a single or staged fashion. In this case, the patient demonstrates considerable skin laxity and has little extra fat, so a dermatolipectomy alone will address her deformity. A limited medial brachioplasty is useful for patients with skin laxity primarily in the proximal third of the arm and involves resection of a vertical ellipse of skin, leaving the scar in the apex of the axilla. The patient described here has skin laxity throughout the upper arm, which would not be adequately addressed with a medial brachioplasty. Liposuction followed by brachioplasty in a staged fashion will not improve her outcome and will only serve to increase the cumulative risks of two surgeries. Liposuction alone does not address the underlying issue of skin laxity. The patient’s age and her history of obesity/massive weight loss severely decrease the ability of her skin to retract after liposuction.

How well did you know this?
1
Not at all
2
3
4
5
Perfectly
23
Q

A 38-year-old woman comes to the office because of excess fullness of the proximal posterior arm. Pinch test shows a thickness of greater than 2 cm, and skin laxity is not excessive. Liposuction is planned. Which of the following is the most likely complication of liposuction of the upper extremity?

A) Contour irregularities
B) Hyperpigmentation
C) Injury to the ulnar nerve
D) Lymphedema
E) Seroma

A

The correct response is Option A.

The most common complication associated with liposuction of the arm is areas of overresection resulting in contour irregularities. Because such irregularities can be due to postoperative swelling, they should be treated conservatively for at least 6 months after surgery. Treatment may consist of fat grafting if contour abnormalities persist after 6 months. Early treatment may include lymphatic massage.

Although hyperpigmentation is possible, it is more commonly seen in the medial thigh in association with ultrasound-assisted liposuction. Seromas are rare in the upper extremity. Injury to the ulnar nerve at the elbow is possible, but it has not been reported. Care should be taken to avoid the nerve in placement of the cannula. While lymphedema is possible, if liposuction of the arm is limited to the posterior aspect, where major lymphatic channels are avoided, it can be prevented. Transient postoperative swelling is expected.

How well did you know this?
1
Not at all
2
3
4
5
Perfectly
24
Q

A 53-year-old woman comes to the office for evaluation of abdominal skin redundancy. Physical examination shows muscle laxity and lipodystrophy of the central abdomen and flank regions. Abdominoplasty and liposuction of the flank and anterior abdominal skin are planned. After this combined procedure, the patient is at greatest risk for skin necrosis of which of the following abdominal zones?

A) I
B) II
C) III
D) IV

A

The correct response is Option A.

Abdominoplasty in combination with liposuction has been associated with a higher risk of complications. Combining both procedures has been reported to increase the risk of delayed healing, thrombotic emboli, fat emboli, skin necrosis, and fat necrosis. Increased complication rates are reported in patients with risk factors such as obesity, smoking, and diabetes mellitus. Direct undermining of the abdominal skin combined with liposuction can lead to vascular compromise of the overlying skin. The blood supply to the abdominal wall is divided into three zones: zone I, mid abdomen supplied by the deep inferior epigastric artery; zone II, lower abdomen supplied by the external iliac artery; and zone III, lateral abdomen and flanks supplied by the intercostal, subcostal, and lumbar arteries. Zone IV has not been described. Following abdominoplasty, elevation of the abdominal flap disrupts the blood supply from zones I and II, leaving the flap to be perfused by blood vessels from zone III. Liposuction with abdominoplasty of the central abdomen, zone I, is associated with the highest rate of skin necrosis. “Safe zones” include the flanks and upper lateral abdomen.

How well did you know this?
1
Not at all
2
3
4
5
Perfectly
25
Q

A 32-year-old man is brought to the office 24 hours after he underwent liposuction of the upper extremities, breasts, and chest. He appears disoriented and confused. Examination shows a petechial rash over the anterior trunk and axilla. Which of the following is the most likely cause?

A) Allergic reaction to a medication
B) Fat embolism
C) Lidocaine toxicity
D) Thrombotic thrombocytopenic purpura
E) Transient ischemic attack

A

The correct response is Option B.

Fat embolization syndrome (FES) is clinically characterized by a triad of symptoms occurring within 24 to 72 hours following surgery or trauma: 1) alterations in mental status, 2) respiratory dysfunction, including hypoxemia or tachypnea, and 3) a petechial rash involving the anterior trunk, axillary, or head and neck regions.

Common predisposing conditions include traumatic long bone and pelvic fractures, orthopedic procedures, liposuction, and soft-tissue injuries. Non-traumatic etiologies include pancreatitis, diabetes mellitus, osteomyelitis, and alcoholic liver disease. Proposed mechanisms for fat embolism include 1) mechanical disruption—release of fat droplets from disrupted bone marrow or adipose tissue forced into torn venules in areas of trauma, or 2) biochemical—release of free fatty acids as chylomicrons induced by systemic changes from trauma or sepsis.

Diagnosis of FES is primarily clinical. Major diagnostic criteria include respiratory distress, cerebral dysfunction, and petechial rash. Minor criteria include tachycardia, tachypnea, fever, hypoxemia, thrombocytopenia, and hypocalcemia. Treatment for FES is primarily supportive. Respiratory support should focus on maintaining adequate tissue oxygenation and arterial saturation. Mechanical ventilation may be necessary. Hemodynamic and fluid resuscitation may be required. Use of corticosteroids remains controversial.

The triad of symptoms including the petechiae concentrated in the upper truncal region would be an unusual occurrence for an allergic medication reaction. Lidocaine toxicity occurs initially with perioral numbness, vertigo, and visual disturbances, and progresses to muscle twitching, unconsciousness, seizures, and finally to cardiorespiratory failure. Lidocaine toxicity would be earlier in onset and is not associated with a petechial rash. Although thrombotic thrombocytopenic purpura (TTP) may initially occur with petechiae, TTP is more commonly associated with headache, confusion, and digestive symptoms such as diarrhea, nausea, and abdominal pain. Transient ischemic attack may occur with temporary loss of vision, hemiparesis, confusion, or paresthesia.

How well did you know this?
1
Not at all
2
3
4
5
Perfectly
26
Q

A 60-year-old woman comes to the emergency department because of shortness of breath and right-sided chest pain 2 days after she underwent an uncomplicated abdominoplasty. Heart rate is 100 bpm, respiratory rate is 20/min, blood pressure is 110/60 mmHg, and oxygen saturation is 92% on room air. Chest x-ray study shows no abnormalities. Serum creatinine concentration is 2.5 mg/dL. Which of the following is the most appropriate first step in management?

A) CT angiography
B) Emergency cardiac catheterization
C) Enoxaparin therapy
D) Lower extremity venous Doppler
Ventilation/perfusion scanning

A

The correct response is Option E.

The most appropriate first step in management is to obtain a ventilation/perfusion scan. The patient described presents with the clinical picture of a pulmonary embolus (PE). Normal chest X-ray study rules out an obvious pulmonary process like pneumonia, atelectasis, or pneumothorax. The right-sided strain pattern on electrocardiography points to a PE as well. Vitals are abnormal, but she is hemodynamically stable. CT angiography (CTA) would have been the investigation of choice; however, serum creatinine is abnormally high, which precludes giving her the high doses of contrast dye a CTA would require. Empirically starting her on anticoagulation is another management option, though one has to weigh the risk-benefit of full-anticoagulation therapy so soon after major surgery. However, subcutaneous enoxaparin therapy in a patient with renal insufficiency is not ideal. Instead, systemic heparin therapy would be the agent of choice because it is not dependent on renal excretion, has a much shorter half-life, and its therapy can easily be measured with activated partial thromboplastin time levels. Therefore, it can be titrated more reliably, and its anticoagulation effects can be shut off quickly if the need arises.

A venous Doppler examination of the lower extremities will probably need to be done, but it is not the first step in management. Additionally, it does not diagnose a PE. An emergent cardiac catheterization is not indicated because this is not an acute myocardial ischemic event.

A ventilation/perfusion scan is performed in situations where an angiogram (conventional or CTA) is not indicated or available. It is especially useful in patients with renal insufficiency because the perfusion portion does not utilize contrast that could negatively affect the kidneys, but rather uses a radioactive tracer. The ventilation/perfusion scan helps stratify the probability of a patient having a PE, and thus helps the clinician to choose the appropriate way to potentially treat the patient.

How well did you know this?
1
Not at all
2
3
4
5
Perfectly
27
Q

An otherwise healthy 34-year-old woman calls the office because of nonspecific left-sided chest pain and mild shortness of breath 2 weeks after she underwent abdominoplasty with plication of a rectus diastasis and liposuction of the flanks. The procedure was performed during general anesthesia in 3 hours and 15 minutes. She was admitted overnight, and heparin was administered subcutaneously during her hospital stay, including one dose preoperatively. She reports no postoperative complications and feels extremely well otherwise. She says she is “finally getting back to herself physically” and does not want to worry her husband or children. She just wanted to make sure she had not “pulled a muscle or injured anything.” Which of the following is the most appropriate action by the plastic surgeon?

A) Evaluation at the emergency department
B) Reassurance
C) Reduction of physical activity
D) Use of incentive spirometer at home to assist in improving pulmonary toilet
E) Use of over-the-counter anti-inflammatory drugs

A

The correct response is Option A.

This patient is likely suffering from a pulmonary embolism and requires emergent workup and treatment with anticoagulation. Although she was treated perioperatively with subcutaneous heparin, deep venous thrombosis (DVT) can still develop as well as a resultant pulmonary embolism (PE). In one survey of board-certified plastic surgeons, PE was found to be the leading cause of death following liposuction. In aesthetic plastic surgery patients, abdominoplasty patients appear to be at a higher risk of venous thromboembolism (VTE) when compared to those undergoing other commonly performed elective procedures. Rates range from 1 in 1000 to 1 in 300 for standard abdominoplasty, and when combined with other procedures, the risk of VTE and death from PE increases significantly. Any possible symptoms that are suspicious for PE, even several weeks postoperatively, must be taken very seriously and treated urgently with immediate presentation to the closest emergency department for aggressive and urgent medical workup as well as imaging and laboratory studies to evaluate for DVT and/or PE.

Venous thrombus formation occurs secondary to a triad of factors describes by Virchow: venous stasis, vascular injury, and hypercoagulability. During abdominoplasty, the combination of general anesthesia, supine positioning, and immobilization promotes venous stasis. In addition, decreased venous return prevents clearance of activated clotting factors, further leading to thrombus formation. The highest risk period for fatal postoperative PE occurs 3-7 days after surgery, with approximately 10% of symptomatic PE being fatal within 1 hour of first symptoms. The risk of symptomatic venous thromboembolism is highest within 2 weeks of surgery and remains elevated for 2 to 3 months. Two thirds of patients with a DVT may appear clinically silent making the need for thromboprophylaxis extremely important. Current strategies to reduce risk include proper patient positioning, early ambulation postoperatively, flexion of the patient’s knees 5 degrees, supplemented with mechanical (e.g., pneumatic compression boots) and/or pharmacological prophylaxis in most patients. The most common practice for pharmacological prophylaxis is to begin therapy preoperatively and continue treatment until 5 to 10 days after surgery. Some studies have even suggested a full month of postoperative treatment for those patients at a higher risk.

How well did you know this?
1
Not at all
2
3
4
5
Perfectly
28
Q

A 45-year-old woman comes to the office because of persistent dysesthesias of the medial forearm 2 months after she underwent brachioplasty for brachial ptosis. Examination shows a well-healed scar on the medial aspect of the arm extending to the proximal elbow. Which of the following nerves was most likely injured during the procedure?

A) Axillary
B) Medial antebrachial cutaneous
C) Median
D) Posterior interosseous
E) Ulnar

A

The correct response is Option B.

When performing brachioplasty, injury to the medial antebrachial cutaneous nerve can occur. This nerve runs along to the medial epicondyle and adjacent to the basilic vein. Maintaining superficial dissection at the ulnar aspect of the elbow and preserving the basilic vein is paramount in minimizing injury to the medial antebrachial cutaneous nerve.

Although theoretically possible, injury to the ulnar and median nerves has not been reported with brachioplasty. The axillary nerve and posterior interosseous nerve should not be in the field of dissection when performing a brachioplasty.

How well did you know this?
1
Not at all
2
3
4
5
Perfectly
29
Q

A 48-year-old man comes to the office because he is dissatisfied with the appearance of his “enlarged” breasts. He underwent gastric bypass surgery 2 years ago followed by a 110-lb (50-kg) weight loss. Current weight is 185 lb (84 kg), which has been stable for 6 months. Physical examination shows nipples with an enlarged areola inferior to the inframammary fold, excess skin with loss of skin elasticity, and a moderate amount of fatty tissue. No palpable or painful masses are noted. Which of the following is the most appropriate surgical procedure for correction of this patient’s deformity?

A) Injection of phosphatidylcholine
B) Nonsurgical radiofrequency fat ablation
C) Reduction mammaplasty
D) Skin-sparing mastectomy
E) Ultrasound-assisted liposuction

A

The correct response is Option C.

This patient suffers from pseudogynecomastia, also known as lipomastia.

Reduction mammaplasty with repositioning the nipple at or above the inframammary fold, reduction of the size of the areola, removal of excess skin, and removal of excess fatty tissue will most appropriately correct this patient’s deformity. However, many insurance companies recognize this code as a gender-specific code for women. This patient’s concerns are primarily with appearance. Breast reduction should be performed as a cosmetic procedure.

No good controlled studies show the benefit of radiofrequency in the treatment of gynecomastia or pseudogynecomastia. The safety and efficacy of phosphatidylcholine injections have yet to be established. Liposuction will not address the skin problem or correct the enlarged areolae. Mastectomy for gynecomastia is a surgical procedure for the removal of painful, periareolar glandular tissue usually in pubescent males and is sometimes covered by insurance.

How well did you know this?
1
Not at all
2
3
4
5
Perfectly
30
Q

A 32-year-old woman, gravida 3, para 3, comes to the office for consultation regarding a slimmer abdominal contour. She says the most weight she lost after pregnancy was 30 lb (13.6 kg). Current BMI is 23 kg/m2. Examination shows mild diastasis recti and vertical skin redundancy above and below the umbilicus. No hernias or scars are noted. Which of the following is the most appropriate treatment?

A) Suction-assisted lipectomy
B) Repair of the diastasis with prosthetic mesh and cryolipolysis
C) Miniabdominoplasty and suction-assisted lipectomy of the epigastrium
D) Abdominoplasty with rectus plication
E) Fleur-de-Lis abdominoplasty

A

The correct response is Option D.

In addition to a thorough medical history, careful analysis of the patient’s goals and evaluation of the anatomy is the basis for correctly selecting the optimum choice for aesthetic restoration of the abdomen after child bearing. This patient’s skin redundancy above and below the navel requires that panniculectomy incorporate downward tension for the portions superior to and inferior to the umbilicus. Miniabdominoplasty fails to include supraumbilical skin tensioning. Liposuction alone may actually worsen the degree of skin laxity by deflating fat compartments or disrupting musculocutaneous suspensory fasciae. It is unlikely that prosthetic material will be necessary to repair a diastasis when plication (folding adjacent fascial halves with sutures) suffices unless a concurrent hernia of significant proportions exists. Traditional abdominoplasty incorporates mobilization of the abdominal panniculus, addresses laxity above and below the navel, and may incorporate more recent techniques such as progressive tension sutures to ensure an optimum aesthetic result while minimizing the need for revision. Fleur-de-Lis abdominoplasty involves a vertical incision and is not indicated. Cryolipolysis is not effective in this level of skin laxity.

How well did you know this?
1
Not at all
2
3
4
5
Perfectly
31
Q

A 60-year-old woman comes to the office for evaluation of a poor outcome after liposuction of the arms. She says she is dissatisfied with the marked laxity of the skin of her arms. A photograph is shown. History includes gastric bypass surgery followed by a 100-lb (45.3-kg) weight loss. BMI is 28 kg/m2. Which of the following procedures is most appropriate to improve contour of the arm in this patient?

A

The correct response is Option D.

Traditional brachioplasty comprises a T-shaped scar along the length of the arm and the axilla. This patient has excess skin on her arm, which dictates the necessity for an excisional procedure. Further liposuction of any modality will exaggerate her presentation further, including Vaser liposuction, a form of ultrasound-assisted liposuction, and laser-assisted liposuction, both of which are reported to tighten skin through heating its undersurface. There is no evidence to support Thermage as an effective treatment for lax skin. Lipobrachioplasty is a technique which safely combines liposuction with excisional surgery, performing liposuction on the proposed area to be excised. In a thinned arm, potential risks of neuropathy and seroma that accompany liposuction are unnecessary in a patient with lax skin that requires excision to improve contour. Limited incision brachioplasty with excision limited to an ellipse in the axilla, with or without liposuction, provides very limited results with regard to improving overall arm contour, particularly with moderate-to-severe skin redundancy.

How well did you know this?
1
Not at all
2
3
4
5
Perfectly
32
Q

A 21-year-old female cheerleader is scheduled to undergo suction-assisted lipectomy for contouring of the outer thigh. She is physically fit and has lost 10 lb (4.5 kg) over the past 6 months. Current weight is 145 lb (65.8 kg). Preoperative examination showed adiposity along the outer thigh area. To decrease the risk of postoperative deformities in this patient, particular attention should be paid to which of the following zones of adherence?

A) Gluteal crease
B) Iliotibial tract
C) Lateral gluteal depression
D) Mid medial thigh
E) Posterior thigh

A

The correct response is Option C.

The key element to avoid postoperative deformities in liposuction is avoiding the zones of adherence. These are zones where the superficial fascial system sends elements through the deep compartment attaching to the investing fascia of the underlying musculature. These zones of adherence accentuate localized fat deposits between them. The lateral gluteal depression is just superior to the outer lateral thigh and attention to that zone is important when performing liposuction along the outer thigh area. The other zones are not related to the outer thigh, except the inferolateral iliotibial tract, which is distal to the area of concern.

How well did you know this?
1
Not at all
2
3
4
5
Perfectly
33
Q

A 30-year-old woman is brought to the emergency department 1 day after undergoing outpatient liposuction because of difficulty breathing and confusion. A tumescent technique with lidocaine was used during the procedure, in which 4000 mL of tumescent fluid was infiltrated and 4000 mL of lipoaspirate was removed. Physical examination shows respiratory distress and petechial rash. Which of the following is the most likely cause of this patient’s condition?

A) Drug allergy
B) Fat embolism
C) Lidocaine toxicity
D) Pulmonary edema
E) Pulmonary embolism

A

The correct response is Option B.

A known risk of liposuction is fat embolism syndrome (FES), and clinical examination remains the gold standard for diagnosis. The three classic symptoms of FES are respiratory distress, decreased cerebral function, and petechial rash. This generally occurs within 48 hours postoperatively. With manual disruption of both fatty tissue and blood vessels that occurs with liposuction, microparticulate fat showers the lung, brain, kidney, and skin with emboli, leading to mechanical obstruction and/or a biochemical inflammatory reaction. Overall mortality from FES after liposuction is approximately 10 to 15%.

Lidocaine at high concentrations can cause serious central nervous system disturbances, including anxiety, agitation, psychosis, seizures and/or coma, and cardiovascular toxicity, including arrhythmias and hypotension. Symptoms of toxicity include light-headedness, euphoria, digital paresthesia, restlessness, and drowsiness. Symptoms of objective toxicity include nausea, vomiting, tremors, blurred vision, tinnitus, confusion, excitement, psychosis, and muscular fasciculations. Seizures and cardiorespiratory depression occur typically when blood levels reach 8 to 12 mcg/mL. Above this level, subjects may become comatose, with respiratory arrest and cardiac asystole. A rash is not associated with lidocaine toxicity. The safe dose of lidocaine has been established as 35 mg/kg during liposuction, and some authors suggest using 55 mg/kg may be safe.

Anaphylaxis from a drug allergy also does not cause a rash but can cause respiratory distress from laryngeal edema.

Pulmonary edema is characterized by respiratory distress that does not respond to oxygen therapy. This can occur because of fluid overload or from physiologic issues related to extubation.

Pulmonary embolism must be considered with respiratory distress but is not associated with rash and can occur at any time postoperatively.

How well did you know this?
1
Not at all
2
3
4
5
Perfectly
34
Q

A 43-year-old woman is scheduled to undergo mastopexy and circumferential lower body lift in an accredited hospital setting for body contouring. She lost 150 lbs (68 kg) after undergoing bariatric surgery 3 years ago. The planned duration of the procedure is 6 hours. In the preoperative holding room, she is dressed in an air-heated gown. Maintenance of normothermia during the perioperative period is most likely to decrease the risk of which of the following complications in this patient?

A)Deep venous thrombosis
B)Fat necrosis
C)Pulmonary embolism
D)Seroma formation
E)Surgical site infection

A

The correct response is Option E.

There is a significant and growing body of evidence showing that normal body temperature during surgery specifically reduces the likelihood of surgical site infections and reduces the risk of bleeding. For this patient with a long operative time and exposure of a significant amount of body surface area, she is at increased risk for hypothermia and surgical site infection.

Fat necrosis is attributable to devascularization of fatty tissue. Seroma formation is attributable to inadequate drainage of the surgical wounds.

The likelihood of deep venous thrombosis and pulmonary embolus is reduced by the use of subcutaneous heparin, low-molecular-weight heparin, and/or sequential compression devices.

How well did you know this?
1
Not at all
2
3
4
5
Perfectly
35
Q

A 50-year-old woman is scheduled to undergo a cosmetic procedure in an ambulatory surgery center (ASC). The patient is concerned about her risk of dying during this procedure. This patient?s postoperative risk of death from a pulmonary embolism is highest if she undergoes which of the following procedures in an ASC?

A) Abdominoplasty
B) Augmentation mammaplasty
C) Rhinoplasty
D) Rhytidectomy
E) Suction lipectomy

A

The correct response is Option A.

Abdominoplasty is the procedure associated most frequently with postoperative mortality from a pulmonary embolism in an ASC.

The American Association for Accreditation of Ambulatory Surgery Facilities, Inc. (AAAASF) has collected statistics on morbidity and mortality for its accredited facilities. Mortality from office-based surgery is extremely rare. From 2001 to 2006, there were over 1 million outpatient procedures performed with 23 deaths. Pulmonary embolism was the cause in 13 of the 23 deaths. Twelve of the 13 pulmonary embolism deaths were associated with abdominoplasty alone or in conjunction with another procedure.

How well did you know this?
1
Not at all
2
3
4
5
Perfectly
36
Q

A 46-year-old woman scheduled to undergo abdominoplasty and lipoplasty is concerned about postoperative discomfort. A regional block during the procedure is planned for pain management to minimize the need for narcotics. The anesthetic for the regional block should be injected to which of the following tissue planes?

A) Between the external oblique and the internal oblique
B) Between the internal oblique and the transverse abdominis
C) Between the skin and the external oblique
D) Between the transverse abdominis and the transverse fascia
E) Between the transverse fascia and the peritoneum

A

The correct response is Option B.

The thoracolumbar nerves that innervate the anterior abdominal wall travel as multiple mixed segmental nerves that branch and communicate widely in the transverse abdominis plane (TAP), located between the internal oblique and transverse abdominis muscles.

The TAP regional block provides abdominal wall analgesia by affecting the sensorial afferent nerves of T6 to L1 found in this plane. Local anesthetic injected into the TAP has been shown to decrease the amount of postoperative narcotic requirements in abdominal wall surgery.

How well did you know this?
1
Not at all
2
3
4
5
Perfectly
37
Q

An otherwise healthy 38-year-old woman with a BMI of 34 kg/m2 is scheduled to undergo suction lipectomy of the abdomen, hips, and thighs using a tumescent technique and a 5000-mL lipoaspirate. The infiltrate contains lidocaine and epinephrine. Which of the following adjustments to the planned procedure is most appropriate to decrease the risk of intraoperative complications?

A) Decreased rate of infusion of the wetting solution
B) Infiltration of all areas simultaneously
C) Removal of the epinephrine from the wetting solution
D) Substitution of bupivacaine for lidocaine
E) Use of a superwet technique instead of tumescent

A

The correct response is Option E.

In the tumescent technique of infiltration of wetting solution, the tissues are infiltrated with roughly a 3:1 ratio of wetting solution to lipoaspirate. This is generally done to effect, with the tissues becoming firmer once engorged with fluid. In large-volume liposuction (greater than 4 L), it is especially important to understand the possible side effects of the constituent parts of the wetting solution, including the volume of administration, the local anesthetic, and the epinephrine.

In terms of lidocaine toxicity, there are various factors to take into consideration, including the level and rate of drug absorption, drug interactions, fluid management, prothrombogenic factors, and volume of the wetting solution and aspirate. According to the ASPS Safety Committee Advisory on Liposuction, two options are available to decrease the risk of lidocaine toxicity in large-volume liposuction cases: 1) decrease the concentration of lidocaine in the wetting solution; and 2) use smaller volumes of infiltrate by applying the superwet technique (1:1 ratio of infiltrate: lipoaspirate) rather than the tumescent technique. Furthermore, one can omit the lidocaine altogether from the infiltration solution. The pressure and rate of infusion of the wetting solution do not affect the rate of lidocaine absorption. Infiltration of all areas simultaneously will be counterproductive. The use of epinephrine in wetting solutions is critical because it causes vasoconstriction, which results in both improved hemostasis as well as delayed absorption of the anesthetic agent, which prolongs its effect, decreases the amount of anesthetic needed, and reduces the risk of lidocaine toxicity. Therefore, its removal from the wetting solution is not indicated. However, in large-volume liposuction cases, staged infiltration of multiple anatomical sites may provide a wider safety margin.

The use of bupivacaine and prilocaine in wetting solutions has not been clinically studied or assessed. Bupivacaine should be used with caution if added to infiltrate solutions because of its slow elimination and reversal and its potential for severe side effects involving the cardiovascular, neurologic, and hematologic systems.

How well did you know this?
1
Not at all
2
3
4
5
Perfectly
38
Q

A 29-year-old woman comes to the office for evaluation of upper arm fullness. Physical examination shows moderate excess subcutaneous fat and minimal skin laxity. Circumferential liposuction is planned. Which of the following is the most likely complication after liposuction of the bicipital groove region?

A) Compartment syndrome
B) Contour deformity
C) Paresthesia
D) Seroma
E) Skin necrosis

A

The correct response is Option B.

When performing suction lipoplasty of the upper arm, the medial portion around and below the area of the bicipital groove should be avoided. The fat layer in this area is very thin, and wrinkling and poor skin contraction may occur. The most common areas of liposuction of the upper arm include the posterior brachial and the lateral aspects. The treatment of the para-axillary region, the deltoid bulge, and the upper back were popularized in the 1980s and the 1990s and are still performed today.

How well did you know this?
1
Not at all
2
3
4
5
Perfectly
39
Q

A 42-year-old man comes to the office because of numbness and pain near the elbow 1 year after undergoing bilateral L-brachioplasty following a 150-lb (68-kg) weight loss. Current weight is 200 lb (90 kg) and BMI is 32 kg/m2. Nerve electrical conduction studies are most likely to demonstrate injury to which of the following sensory nerves?

A) Lateral antebrachial cutaneous
B) Medial antebrachial cutaneous
C) Posterior antebrachial cutaneous
D) Radial dorsal cutaneous
E) Ulnar dorsal cutaneous

A

The correct response is Option B.

How well did you know this?
1
Not at all
2
3
4
5
Perfectly
40
Q

A 46-year-old woman comes to the office for consultation regarding abdominal contouring surgery 18 months after gastric bypass surgery. She has lost 125 lb (57 kg). Physical examination shows excess abdominal, flank, and back skin. Which of the following nutritional parameters is most likely abnormal in this patient at this time?

A)Albumin
B)Calcium
C)Folate
D)Iron
E)Vitamin B12 (cobalamin)

A

The correct response is Option D.

Following gastric bypass, many patients have poor nutrition that may adversely affect post-bariatric surgical outcomes. Advanced age, changes in body mass index, and presence of dumping syndrome may contribute to the patient’s overall nutritional status. Preoperative assessment is important in identifying patients with inadequate nutrition. Evaluation should include protein-calorie intake, serum protein measures, vitamin and mineral status, coagulation, liver function, and electrolytes.

Iron deficiency is the most common abnormality found in post-gastric bypass patients. Despite iron supplementation, 50% of patients show evidence of low ferritin concentrations. Vitamin B12 (cobalamin) deficiency is often detected. Deficiencies in calcium and folate are less common.

Protein deficiency identified by hypoalbuminemia has been reported to be as high as 14% in patients following gastric bypass surgery.

How well did you know this?
1
Not at all
2
3
4
5
Perfectly
41
Q

A 35-year-old woman comes to the office 2 weeks after undergoing abdominoplasty. She says she has felt bloated for the past week, and that she expects her menstrual period in 2 days. Physical examination shows ballottable swelling of the infraumbilical region. Which of the following is the most appropriate next step in management?

A) Application of a compression garment
B) Diagnostic ultrasonography
C) Doxycycline sclerosis
D) Needle aspiration
E) Oral administration of a diuretic agent

A

The correct response is Option D.

Seroma formation is the most frequent complication of abdominoplasty procedures. The occurrence rate quoted in the literature varies from 0.3 to 90%. The most common figure is 7 to 35%. The diagnosis of seroma can usually be made on physical examination. Ballottable swelling or fluid wave is both visible and palpable and represents the sine qua non of seroma.

In this patient, a needle aspiration should be performed. It is not unusual for seroma to recur, and additional aspirations may be required on a weekly basis. The average seroma takes two to three aspirations to eradicate. If seroma is ignored, it can lead to a permanent condition that requires operative excision. Risk factors for increased seroma formation include BMI greater than or equal to 30, concomitant liposuction, large skin resections, and shear forces. Most plastic surgeons put their patients in compression garments postoperatively; however, studies have shown that garments do not prevent seromas.

Diagnostic ultrasound is an unnecessary test when the patient has a ballottable mass. Doxycycline sclerosis is indicated when aspiration is unsuccessful or if the suction drains cannot be removed because of persistent high-volume output.

Oral diuretics will not cure seroma.

How well did you know this?
1
Not at all
2
3
4
5
Perfectly
42
Q

A 40-year-old woman comes to the office because she is dissatisfied with the “deflated” appearance of her buttocks (shown) after undergoing gastric bypass surgery, followed by a 100-lb (45-kg) weight loss. Physical examination shows skin laxity and deflation of the buttocks. In addition to performing a lower body lift, which of the following is the most appropriate management of the buttock deformity?

A) Application of external radiofrequency (Thermage)
B) Augmentation with gluteal artery perforator flap coverage
C) Fat grafting
D) Implantation of prostheses
E) Suction lipectomy

A

The correct response is Option B.

Gluteal changes related to loss of volume, ptosis, and skin laxity following massive weight loss require aggressive surgical techniques to improve contour and augmentation.

External radiofrequency (Thermage) does not address deflation and has shown conservative results with regard to skin tightening. It does not offer the degree of tightening that would adequately treat the massive weight-loss patient. The gluteal region in patients who have sustained massive weight loss following gastric bypass surgery is characterized by excessive skin and exaggerated fat loss. Lower body lift procedures remove excess skin and lift sagging buttock tissue, but they do not address deflation and may result in further gluteal flattening. Autologous gluteal augmentation flaps, such as those vascularized by the superior and inferior gluteal artery perforator arteries, can be transferred reliably during lower body lifts to add volume and projection to the buttocks.

Fat grafting is the most prevalent form of gluteal augmentation with the greatest safety profile, currently surpassing gluteal prostheses. The greatest challenge in using autologous fat in a patient who has sustained massive weight loss is assessing whether the patient has adequate donor fat to overcome severe volume loss and skin laxity. The amount of donor fat needed to achieve a pleasing shape is much greater than it seems, ranging from 450 to 1100 mL per side, depending on patient size and gluteal dimension. The ideal candidate for autologous buttock augmentation is slightly overweight, but in good health.

Implantation of gluteal prostheses is not often performed by plastic surgeons due to well-known risks that include wound dehiscence, extrusion, seroma, and infection. Furthermore, gluteal prostheses carry risks of rotation, capsular contracture, and displacement.

Suction lipectomy would exacerbate volume deficiency and result in little to no skin tightening.

How well did you know this?
1
Not at all
2
3
4
5
Perfectly
43
Q

A 42-year-old woman is undergoing brachioplasty after a massive weight loss. The surgeon is most likely to avoid injury to the medial antebrachial cutaneous nerve during the procedure if he or she is able to identify it in the elbow at which of the following locations?

A) Deep to the deep fascia traveling with the basilic vein
B) Deep to the deep fascia traveling with the cephalic vein
C) Superficial to the deep fascia traveling with the basilic vein
D) Superficial to the deep fascia traveling with the cephalic vein

A

The correct response is Option C.

The position of the medial antebrachial cutaneous nerve (MABC) makes it vulnerable to injury during a brachioplasty. Understanding its course and relationship to other structures allows the surgeon to identify and protect it. The MABC is a sensory nerve. It is a terminal branch of the medial cord of the brachial plexus. It travels with the basilic vein medial to the biceps brachii muscle, and it pierces the fascia at the basilic hiatus proximal to the elbow. Injury to the MABC can cause areas of numbness of the median forearm, as well as painful neuromas. The MABC is best identified in the distal aspect of the brachioplasty incision with the basilic vein and protected during the dissection.

How well did you know this?
1
Not at all
2
3
4
5
Perfectly
44
Q

An otherwise healthy 35-year-old man comes to the office because of a 15-year history of bilateral Grade III gynecomastia. Height is 5 ft 10 in (178 cm) and weight is 187 lb (85 kg). BMI is 27 kg/m2. Physical examination shows breast enlargement with skin redundancy and palpable glandular and fatty tissue. Ultrasound-assisted lipectomy is planned because the patient wishes to avoid visible scars. Which of the following is most likely to minimize the need for skin resection?

A)Disruption of the inframammary fold
B)High energy setting
C)Peripheral feathering with standard liposuction
D)Treatment of the intermediate fat layer
E)Tumescent infiltration with a 3:1 ratio

A

The correct response is Option A.

Although surgery remains the mainstay of treatment for gynecomastia in this country, results have been inconsistent, with reported dissatisfaction rates as high as 50%. Common aesthetic problems reported after surgery include unacceptable scarring and nipple and areolar deformities such as tethering and malposition. As a result, less invasive techniques have evolved and are now the mainstay of treatment for all but the most severe forms of gynecomastia.

The use of ultrasound-assisted lipectomy has improved the outcomes in the treatment of all grades of gynecomastia. In the more severe grades, certain maneuvers will improve skin retraction and redraping and may thus eliminate the need for a subsequent procedure for skin excision. While fatty and fibrous tissue is best approached from a deeper subcutaneous plane, transition to a subdermal plane can allow for greater skin retraction. Moreover, disruption of the inframammary fold is essential in that it allows the skin to drape more naturally onto the abdomen, which is typical of a male breast.

The use of wetting solutions is imperative with ultrasound-assisted lipectomy to avoid burns. Both superwet (1:1) and tumescent (3:1) techniques are acceptable. The power of the unit should generally not exceed 90% to avoid burns and dermal injuries. Peripheral feathering improves contour, providing a smoother transition to the outer border of the breast, but does not specifically address skin excess.

How well did you know this?
1
Not at all
2
3
4
5
Perfectly
45
Q

A 40-year-old woman comes to the office because she is dissatisfied with the “deflated” appearance of her buttocks (shown) after undergoing gastric bypass surgery, followed by a 100-lb (45-kg) weight loss. Physical examination shows skin laxity and deflation of the buttocks. In addition to performing a lower body lift, which of the following is the most appropriate management of the buttock deformity?

A) Application of external radiofrequency (Thermage)
B) Augmentation with gluteal artery perforator flap coverage
C) Fat grafting
D) Implantation of prostheses
E) Suction lipectomy

A

The correct response is Option B.

Gluteal changes related to loss of volume, ptosis, and skin laxity following massive weight loss require aggressive surgical techniques to improve contour and augmentation.

External radiofrequency (Thermage) does not address deflation and has shown conservative results with regard to skin tightening. It does not offer the degree of tightening that would adequately treat the massive weight-loss patient. The gluteal region in patients who have sustained massive weight loss following gastric bypass surgery is characterized by excessive skin and exaggerated fat loss. Lower body lift procedures remove excess skin and lift sagging buttock tissue, but they do not address deflation and may result in further gluteal flattening. Autologous gluteal augmentation flaps, such as those vascularized by the superior and inferior gluteal artery perforator arteries, can be transferred reliably during lower body lifts to add volume and projection to the buttocks.

Fat grafting is the most prevalent form of gluteal augmentation with the greatest safety profile, currently surpassing gluteal prostheses. The greatest challenge in using autologous fat in a patient who has sustained massive weight loss is assessing whether the patient has adequate donor fat to overcome severe volume loss and skin laxity. The amount of donor fat needed to achieve a pleasing shape is much greater than it seems, ranging from 450 to 1100 mL per side, depending on patient size and gluteal dimension. The ideal candidate for autologous buttock augmentation is slightly overweight, but in good health.

Implantation of gluteal prostheses is not often performed by plastic surgeons due to well-known risks that include wound dehiscence, extrusion, seroma, and infection. Furthermore, gluteal prostheses carry risks of rotation, capsular contracture, and displacement.

Suction lipectomy would exacerbate volume deficiency and result in little to no skin tightening.

How well did you know this?
1
Not at all
2
3
4
5
Perfectly
46
Q

A 42-year-old woman is undergoing brachioplasty after a massive weight loss. The surgeon is most likely to avoid injury to the medial antebrachial cutaneous nerve during the procedure if he or she is able to identify it in the elbow at which of the following locations?

A) Deep to the deep fascia traveling with the basilic vein
B) Deep to the deep fascia traveling with the cephalic vein
C) Superficial to the deep fascia traveling with the basilic vein
D) Superficial to the deep fascia traveling with the cephalic vein

A

The correct response is Option C.

The position of the medial antebrachial cutaneous nerve (MABC) makes it vulnerable to injury during a brachioplasty. Understanding its course and relationship to other structures allows the surgeon to identify and protect it. The MABC is a sensory nerve. It is a terminal branch of the medial cord of the brachial plexus. It travels with the basilic vein medial to the biceps brachii muscle, and it pierces the fascia at the basilic hiatus proximal to the elbow. Injury to the MABC can cause areas of numbness of the median forearm, as well as painful neuromas. The MABC is best identified in the distal aspect of the brachioplasty incision with the basilic vein and protected during the dissection.

How well did you know this?
1
Not at all
2
3
4
5
Perfectly
47
Q

A 65-year-old woman comes for evaluation of persistent sharp, stabbing pain in the lower abdomen that radiates to the groin. She underwent abdominoplasty 6 months ago, and the pain first occurred 1 month after the procedure. The patient says that the pain is aggravated when she walks. Weight is 154 lb (70 kg). Which of the following is the most appropriate initial step in management?

A) Electromyography and nerve conduction studies
B) Initiation of a 6-month course of gabapentin
C) MRI of the abdomen and pelvis
D) Nerve block with lidocaine
E) Surgical exploration and excision of neuroma

A

The correct response is Option D.

Although nerve injury is not the most common complication of abdominoplasty, it does occur in approximately 1% of patients undergoing cosmetic procedures and in 2 to 4% of patients who have similarly located Pfannenstiel incisions for obstetric and/or gynecologic procedures. Patients with symptoms of neuropathic pain after abdominoplasty are often referred for a series of diagnostic tests and consultations that may not be necessary, delaying definitive treatment.

Patients experiencing neuropathic pain after abdominoplasty have most often sustained an injury to the lateral femoral cutaneous, iliohypogastric, or ilioinguinal nerve. The typical symptom is pain that can occur immediately or develop over time. Patients usually report a stabbing pain at the corner of the incision that may radiate to the pubic area and/or upper leg. Hyperextension or ?twisting? upper body movements can aggravate the pain. If abdominal muscle contraction intensifies the pain (Carnett sign), the pain is probably located in the abdominal wall. When nerve entrapment is suspected, a nerve block using a short-acting anesthetic injected into the trigger point is the next step in diagnosis. MRI and nerve conduction studies are rarely useful in this setting. If the patient reports relief with the injection, a second block with corticosteroids may provide long-term relief. If not, a well-targeted exploration with possible neurectomy and replantation may be indicated. While treatment with gabapentin (Neurontin) may provide temporary relief, its use is poorly tolerated in the long term and does not aid in the diagnosis or the definitive treatment of the problem.

How well did you know this?
1
Not at all
2
3
4
5
Perfectly
48
Q

Which of the following is more likely to occur with internal ultrasound-assisted lipoplasty when compared with traditional suction-assisted lipectomy?

A) Contour deformities
B) Infection
C) Seroma
D) Skin laxity
E) Surgeon fatigue

A

The correct response is Option C.

Internal ultrasound-assisted lipoplasty (UAL) has been associated with an increased incidence of seroma formation, tissue damage, and thermal damage, as well as neurapraxia. With the use of a cannula or solid probe, ultrasound energy is transmitted to the fat layer, where it disrupts the fat cell. The ?liquefied? fat is then aspirated with suction. To prevent thermal injuries to the skin, infiltration of solution is mandatory, and the cannula or probe needs to be in constant motion.

Although there are unique advantages and disadvantages of each lipoplasty technique, in experienced hands, excellent results can be achieved with any of the techniques, including suction-assisted lipectomy, power-assisted lipoplasty, UAL, and laser-assisted lipoplasty. Contour deformities are related to how the operator performs the technique rather than the technique itself.

Infection rates are similar for the two procedures. Some studies have noted a decrease in both skin laxity and surgeon fatigue with UAL.

How well did you know this?
1
Not at all
2
3
4
5
Perfectly
49
Q

A 27-year-old woman comes to the office for consultation regarding liposuction of the abdomen, hips, flanks, and thighs. Height is 5 ft 5 in (165 cm) and weight is 156 lb (71 kg). BMI is 26 kg/m2. Physical examination shows lipodystrophy of the abdomen, hips, flanks, and thighs. Aspiration of 4 L using suction-assisted lipectomy and a superwet technique is planned. Which of the following is the most likely complication in this patient?

A) Contour irregularities
B) Deep venous thrombosis
C) Hyperpigmentation
D) Seroma
E) Wound infection

A

The correct response is Option A.

Contour irregularity is reported to occur in 20% of cases and is the most common complication in liposuction. Deep venous thrombosis is reported in less than 1% of patients. Cutaneous hyperpigmentation is reported in about 4% of patients. It is often related to friction burns or cutaneous compromise. Seromas are relatively uncommon and are believed to occur more frequently in patients with an increased body mass index. Finally, wound infections are rare following liposuction.

50
Q

A 55-year-old woman comes to the office for consultation about arm contouring. She underwent gastric bypass surgery 18 months ago, followed by a 100-lb (45-kg) weight loss. She says she has difficulty finding clothing that fits her upper arms. A photograph is shown. Physical examination shows skin laxity and lipodystrophy of the upper arms. Brachioplasty with a T-scar and posterior liposuction is planned. Which of the following complications is most likely to occur in this patient?

A) Cellulitis
B) Lymphedema
C) Numbness
D) Thromboembolism
E) Wound dehiscence

A

The correct response is Option E.

The most likely risk of a combination brachioplasty liposuction procedure is wound dehiscence. Wound dehiscence has been most commonly reported when liposuction is performed in conjunction with brachioplasty, and most commonly reported with the excisional technique alone. Dehiscence most often occurs in the axilla at the T point in traditional brachioplasty and in the axilla in the L-brachioplasty approach.

Lymphedema also may occur after brachioplasty with or without liposuction and is also often temporary. There is no increased risk with liposuction. Lymphedema occurs because of scarring around the axillary lymph nodes. Cellulitis may also be associated with lymphatic obstruction or interruption in the arm.

Numbness is a common complication following brachioplasty but is often temporary in nature. The medial antebrachial cutaneous nerve is located within the field of surgery in the distal arm and may undergo traction injury or laceration. Thromboembolism is a rare occurrence after brachioplasty.

51
Q

A 40-year-old woman says she has a burning pain in the thigh with movement the day after undergoing abdominoplasty. Which of the following nerves was most likely injured during the procedure?

A) Genitofemoral
B) Iliohypogastric
C) Ilioinguinal
D) Lateral femoral cutaneous
E) Saphenous

A

The correct response is Option D.

In several studies of complications of abdominoplasty, the most common nerve injury was to the lateral femoral cutaneous nerve. Symptoms include anterior and lateral thigh burning, tingling, and/or numbness, all of which increase with standing, walking, or hip extension. Injury or entrapment of the lateral femoral cutaneous nerve is also known as meralgia paresthetica.

The genitofemoral nerve supplies the proximal portion of the thigh about the femoral triangle just lateral to the skin that is innervated by the ilioinguinal nerve. Nerve injury may result from hernia repair, but injury to this nerve is rare.

The iliohypogastric nerve arises primarily from L1. The distribution of the cutaneous sensation of the iliohypogastric nerve is most commonly a small region just superior to the pubis. The iliohypogastric nerve is rarely injured in isolation. Symptoms include burning pain into the inguinal and suprapubic region.

The ilioinguinal nerve arises from the fusion of the T12 and L1 nerve roots and pierces the transversus abdominis and internal oblique muscles. The nerve then supplies sensory branches to the pubic symphysis, the superior and medial aspect of the femoral triangle, and either the root of the penis and anterior scrotum in men or the mons pubis and labia majora in women. The nerve can be injured in abdominoplasty and other lower abdominal incisions. Symptoms include paresthesia of the skin along the inguinal ligament. The sensation may radiate to the lower abdomen. Pain may be localized to the medial groin, the labia majora or scrotum, and the inner thigh.

Saphenous nerve symptoms of entrapment may include a deep aching sensation in the thigh, knee pain, and paresthesias in the cutaneous distribution of the nerve in the leg and foot.

52
Q

A 27-year-old woman comes to the office for consultation regarding mesotherapy for minimal lipodystrophy of the flank regions. She asks about the relative effectiveness of nonsurgical treatment with mesotherapy as compared to suction lipectomy. Which of the following is the most appropriate response?

A) Mesotherapy is as effective as suction lipectomy, and it is approved by the US Food and Drug Administration (FDA)
B) Mesotherapy is as effective as suction lipectomy, but it is not FDA-approved
C) Mesotherapy is not as effective as suction lipectomy, and it is FDA-approved
D) Mesotherapy is not as effective as suction lipectomy, and it is not FDA-approved

A

The correct response is Option D.

Mesotherapy involves the subcutaneous injection of medications as a nonsurgical alternative to suction lipectomy. Currently, mesotherapy is not an effective alternative to suction lipectomy and is not approved by the US Food and Drug Administration (FDA). Although used initially in the treatment of psoriasis, chronic pain, and cellulite, mesotherapy has been advocated as an alternative to suction lipectomy. In mesotherapy, a variety of medications are injected into the mesoderm, the layer of fat and connective tissue below the skin. The most common medications utilized for lipolysis are phosphatidylcholine and isoproterenol. Currently, there are no clinical reports demonstrating the effectiveness of mesotherapy in body contouring.

53
Q

A 53-year-old woman who underwent laparoscopic gastric bypass surgery comes to the office for consultation regarding abdominal contouring. History includes an open cholecystectomy, ventral hernia repair, appendectomy, and caesarean delivery. The presence of which of the following scars on this patient’s abdomen is most likely to result in postoperative wound healing complications?

A) Laparoscopic port scar
B) Pfannenstiel (low transverse) scar
C) Right lower quadrant scar
D) Right subcostal scar
E) Upper midline scar

A

The correct response is Option D.

The blood supply to the abdominal wall arises from the intercostal arteries, the superior and inferior superficial epigastric arteries, and the perforators from the deep superior epigastric arteries through the rectus abdominis muscle. In a traditional abdominoplasty with undermining of the superior flap up to the costal margin, the superficial inferior epigastric arteries and the perforators that arise from the rectus abdominis muscles are divided. This leaves the abdominal flap to survive on the flow from the intercostal vasculature. The subcostal scar from the prior open cholecystectomy is the most likely scar to pose a problem for wound healing in the patient described. This scar has divided the blood flow through the intercostal circulation; thus, flow inferior to the scar may be unreliable.

The port scars are unlikely to cause problems with blood supply due to their small size. The Pfannenstiel, or low transverse, caesarean section scar will require either inclusion within the tissue to be removed or placement within the inferior incision in the old scar line if it is sufficiently low. Placement of the incision above the old scar may impair the blood flow between the two scars. However, even in the scenario described, it is less likely to cause a healing issue because of the transverse orientation of the scar and the lack of undermining between the new incision and old scar. The right lower quadrant (appendectomy) scar will not be an issue, as this will be removed with the tissue resection. The upper midline scar is not as likely to cause a wound healing issue, as circulation from the intercostal vessels from both sides should supply blood flow to the flap on each side of the midline.

54
Q

A 38-year-old woman undergoes suction-assisted lipectomy of the hips, abdomen, and outer thighs. Using a super-wet technique, 3 L of aspirate is obtained. Which of the following is the most accurate estimate of the percentage of fluid infiltrate that remains in the body at the end of the procedure?

A) 10%
B) 30%
C) 50%
D) 70%
E) 90%

A

The correct response is Option D.

The most accurate estimate of infiltrate remaining in the body is 70%. Fluid management is critical in the treatment of suction lipectomy patients. Profound hemodynamic changes occur with increasing significance as the volume of infiltrate and aspirate increases. To avoid fluid overload leading to pulmonary edema, the plastic surgeon must realize that the majority of infiltrate will remain in the patient’s body and be absorbed over time. Fluid requirements should include maintenance fluid (the amount of fluid required to replace normal daily requirements plus deficits related to being nothing-by-mouth), aspirate removed, and fluid infiltrated (70% estimated to be intravascular). These calculations will be vastly different depending on the type of wetting solution used, such as dry technique (no additional infiltrate), wet technique (200 to 300 mL per site), super-wet technique (1 mL of infiltrate for every 1 mL of aspirate), or tumescent technique (3 mL of infiltrate for every 1 mL of aspirate).

55
Q

A 42-year-old man comes to the office because of numbness and pain near the elbow 1 year after undergoing bilateral L-brachioplasty following a 150-lb (68-kg) weight loss. Current weight is 200 lb (90 kg) and BMI is 32 kg/m2. Nerve electrical conduction studies are most likely to demonstrate injury to which of the following sensory nerves?

A) Lateral antebrachial cutaneous
B) Medial antebrachial cutaneous
C) Posterior antebrachial cutaneous
D) Radial dorsal cutaneous
E) Ulnar dorsal cutaneous

A

The correct response is Option B.

The demand for brachioplasty in the United States has increased as the number of patients undergoing bariatric surgery has increased. While brachioplasty is considered a safe and effective method of treating upper arm skin excess, the reported complication rate ranges from 25 to 40%. Most common complications of brachioplasty are considered minor and include seroma, poor scarring, edema, wound dehiscence, and underresection. The most common major complication is cutaneous nerve injury, which can occur in up to 5% of patients.

Medial placement of the brachioplasty incision in the bicipital groove is preferred because the ultimate scar will be hidden when the arm is adducted. Medial placement of the incision may damage the cutaneous nerves that run in this area. Both the medial antebrachial cutaneous nerve and the medial brachial cutaneous nerve arise from the medial cord of the brachial plexus. The medial brachial cutaneous nerve runs with the basilic vein and sends two to four branches to the skin 7 cm proximal to the medial epicondyle. Another three to five branches pierce the fascia to innervate the skin at about 15 cm proximal to the medial epicondyle. The medial antebrachial cutaneous nerve runs in close proximity with the intramuscular septum and penetrates the fascia approximately 14 cm proximal to the medial epicondyle. At this point, the nerve runs superficially and is at risk for injury.

While injuries to motor branches of the median and ulnar nerve have been reported, these branches run deep to the brachial fascia and are not injured unless the intramuscular septum is inadvertently punctured.

56
Q

A 45-year-old woman comes to the office for consultation regarding body contouring 2 years after undergoing gastric bypass surgery. She has lost 120 lb (54 kg) since the procedure was performed. Height is 5 ft 5 in (165 cm), and weight has been stable at 160 lb (72 kg) for 11 months. The patient desires to undergo as few stages as possible. Lower body lift and bilateral mastopexy are scheduled to be performed in a single stage. Which of the following postoperative complications is most likely to occur in this patient?

A) Dehiscence
B) Hematoma
C) Infection
D) Pulmonary embolism
E) Seroma

A

The correct response is Option A.

In appropriately selected patients, multiple body contouring procedures can be combined into a single stage if the surgery can be done in a timely fashion. Overall minor complication rates are higher; however, per procedure complication rates do not seem to increase if more procedures are performed in a single stage. The most common complication in patients undergoing combined procedures in a single stage is related to wound healing issues (approximately 15%). Seroma is the next most common complication (approximately 10%). Infection, hematoma, and pulmonary embolism occur at rates of less than 5%.

57
Q

A 35-year-old woman comes to the office for consultation because she is unhappy with the unevenness of her skin 1 year after undergoing power-assisted suction lipectomy using the super-wet technique. Physical examination shows skin surface irregularities that are radially oriented around a single 1-cm-wide scar. Which of the following modalities was most likely used for this outcome?

A) Improper power source
B) Multiple access sites
C) Poor postoperative compression
D) Superficial plane of suction
E) Use of fine cannulas

A

The correct response is Option D.

One of the most common deformities of suction lipectomy is surface irregularity, which can have several causes. A large cannula will create a large furrow that may be visualized if the suction lipectomy is not performed evenly. Use of a single port may also lead to irregularities because the suctioning is done from only one angle. Superficial suctioning is also more prone to visible irregularities. The best way to avoid these deformities is to use small cannulas in the deep fat, with cross-tunneling from two sites, such that the tunnels are at right angles to each other. The power source would not cause surface irregularities. Lack of compression will cause prolonged edema, but not surface irregularities.

58
Q

A 48-year-old woman is scheduled to undergo abdominoplasty. She has smoked one pack of cigarettes daily for the past 10 years. During the preoperative visit, she informs the plastic surgeon that she has been on a nicotine patch and has not been smoking for 3 weeks. Which of the following mechanisms is most likely to cause wound healing complications in this patient?

A ) Decreased availability of hemoglobin

B ) Decreased red blood cell deformability

C ) Impairment of leukocyte function

D ) Increased fibrinogen production

E ) Increased microvascular vasoconstriction

A

The correct response is Option E.

The nicotine in cigarettes causes vasoconstriction of cutaneous blood vessels with resultant decreased tissue oxygenation. Smoking also increases carboxyhemoglobin, increases platelet aggregation, increases blood viscosity, decreases collagen deposition, and decreases prostacyclin formation, which all negatively affect wound healing. In addition, vasoconstriction associated with smoking is not a transient phenomenon. Smoking a single cigarette may cause cutaneous vasoconstriction for up to 90 minutes; hence, a patient who smokes one pack of cigarettes daily remains tissue hypoxic for most of each day. Vasoconstriction is mediated directly and indirectly by nicotine, a colorless, odorless, and poisonous alkaloid. A reduction in tissue perfusion results from elevated cellular levels of nicotine. The indirect pathways of vasoconstriction include the enhancement of thromboxane A2 and the stimulation of catecholamine release from the adrenal medulla, sympathetic ganglia and nerve endings, and cardiac chromaffin tissue. Other by-products, such as hydrogen cyanide, inhibit the enzymatic pathways vital for cellular oxidative metabolism and oxygen transport, effectively diminishing the ability for cellular repair and wound healing. Combined with acrolein, another toxic gaseous component, hydrogen cyanide inhibits leukocyte function, further impairing the inflammatory phase of healing. The proliferation of macrophages and fibroblasts, which are integral to the phases of wound healing, is also diminished by the presence of nicotine. Additionally, the presence of nicotine and catecholamines stimulates the production of chalones, which are hormones that retard and decrease the rate of wound epithelialization. Collagen deposition is also decreased in smokers. Nicotine is also associated with thrombogenesis by interfering with prostaglandin I2 (prostacyclin) activity. Prostaglandin I2 is a potent vasodilator and inhibitor of platelet aggregation. Platelet adhesiveness is augmented, raising the potential for thrombotic microvascular occlusion and subsequent tissue ischemia.

Carbon monoxide is another toxic by-product common in tobacco smoke. The oxygen-carrying capacity of blood is reduced by the competitive, inhibitory binding of carbon monoxide to hemoglobin. Carboxyhemoglobin levels rise, and tissue delivery of oxygen is reduced as the oxygen-hemoglobin saturation curve is shifted to the left. The decrease in oxygen available for tissue consumption leads to diminished wound healing. The resulting hypoxic state stimulates erythropoiesis, red blood cell aggregation, and fibrinogen production, leading to increased blood viscosity, which potentiates an environment already ripe for thrombogenesis. Decreased red blood cell deformability is also noted in smokers but through an unknown mechanism.

59
Q

A 23-year-old woman comes to the office for consultation regarding significant skin laxity and volume loss in the face, torso, and upper and lower extremities following gastric bypass surgery 22 months ago. She has lost 120 lb (54 kg) since the procedure was performed. Rhytidectomy, abdominoplasty, brachioplasty, back lift, and thigh lift will be performed in three stages. Which of the following procedures will most likely result in hypertrophic scarring?

A ) Abdominoplasty

B ) Back lift

C ) Brachioplasty

D ) Mastopexy

E ) Rhytidectomy

A

The correct response is Option C.

Hypertrophic scarring is reported to occur in up to 40% of brachioplasty cases. These scars require compression therapy, silicone sheeting, or mild corticosteroid injections. The other body regions are less likely to scar in a hypertrophic fashion.

60
Q

A 38-year-old woman comes to the office desiring abdominoplasty. History includes easy bruising and four spontaneous abortions in the first trimester and one live birth. Her infant weighed 10 lb 8 oz (4.5 kg) at birth. The patient’s BMI is 28 kg/m2. Examination shows a 6-cm diastasis recti, an abdominal pannus, and a well-healed Pfannenstiel incision. Which of the following consultations is most appropriate for this patient before proceeding with surgery?

A ) Cardiologist

B ) Endocrinologist

C ) General surgeon

D ) Hematologist

E ) Nutritionist

A

The correct response is Option D.

Venous thromboembolic events (VTEs), which include both deep venous thrombosis and pulmonary emboli, remain a significant cause of morbidity and mortality in surgical patients. The most commonly associated aesthetic procedure is abdominoplasty. For this reason, protocols should be in place to identify patients who are at higher risk. Multiple spontaneous abortions are highly suggestive of a hypercoagulation diathesis and should be further evaluated. The most common genetic abnormality is Factor V Leiden, which is a mutated factor V resistant to inactivation by activated protein C. This abnormality is present in 3 to 7% of the Caucasian population.

Other genetic thrombophilic conditions include prothrombin variant 20210A, antiphospholipid antibodies, protein C or S deficiency, antithrombin deficiency, and hyperhomocysteinemia. The risk of VTE increases greatly when more than one of these abnormalities is present. Hematologic evaluation through blood testing can identify these high-risk patients. Following this evaluation, the hematologist can provide insight regarding the risk of VTE. The decision is then made to use VTE prophylaxis or not to perform the operation at all. The most common protocol is subcutaneous administration of enoxaparin (Lovenox) 40 mg once daily begun 12 hours postoperatively and continued until full ambulation. Protocols vary in both time of initiation of low-molecular-weight heparin and length of treatment.

The patient delivered an infant weighing 10 lb 8 oz (4.5 kg). This occurrence may represent gestational diabetes; however, a more thorough history can identify this possibility.

A trained plastic surgeon should be able to identify the difference between a diastasis recti and a true hernia. Only in extraordinary cases should a general surgeon be required, for example, if the patient has a history of multiple recurrent hernias or a very thin abdominal wall.

Studies have shown that complication rates in abdominoplasty increase in patients with a BMI greater than or equal to 30 kg/m2.

61
Q

A 40-year-old man becomes increasingly disoriented and obtunded 1 day after belt lipectomy. He has had a 200-lb (91-kg) weight loss since undergoing gastric bypass surgery 22 months ago. Which of the following is the most appropriate initial step in management?

A ) Anesthesia consultation

B ) Duplex ultrasonography of the legs

C ) Heparin therapy

D ) Psychiatric evaluation

E ) Thiamine therapy

A

The correct response is Option E.

Many massive weight-loss patients suffer from malnutrition, including thiamine deficiency, which can lead to Wernicke-Korsakoff encephalopathy. Treatment is intravenous administration of 100 mg/d of thiamine, continuing with 100 mg every 8 hours until resolution of symptoms. Administration of thiamine is low risk and may reverse symptoms. CT scan of the head is worthwhile but can wait until after thiamine is administered. It is possible the patient could have suffered a €œkink € in his carotid or vertebrobasilar system with prone positioning, particularly if the head was positioned to the side and the neck was not in neutral position. Duplex ultrasonography of the legs, which can rule out deep venous thrombosis and possible pulmonary embolism, is recommended if there is a high-risk background, but it is unlikely 1 day after surgery. Therapeutic heparin administration would present a danger for postoperative bleeding only 1 day after belt lipectomy. Metabolic or surgical issues should be addressed before a psychiatrist is involved.

62
Q

A 57-year-old woman comes for evaluation for body contouring. She takes no medications except garlic herbal tablets daily. Use of the herbal medication increases her risk for which of the following postoperative events?

A ) Bleeding

B ) Hypertension

C ) Nausea and vomiting

D ) Prolonged anesthesia effects

E ) Tachycardia

A

The correct response is Option A.

Among the general population, garlic is used as a common herbal supplement and, as such, is taken by a significant percentage of surgical patients. Proposed benefits of this supplement include its activity as a potential antibiotic, diuretic, antihypertensive, lipid-lowering agent, and antitussive.

However, garlic ingestion may also increase the risk of intraoperative or postsurgical bleeding secondary to its platelet inhibitor activity or its role as a stimulator of fibrinolytic activity.

Common medication classes that may interact with garlic include anticoagulants, hypoglycemics, cardiovascular medications, and monoamine oxidase inhibitors. Because a number of prescription medications may interact with garlic, and because most patients do not consider herbal substances €œmedications, € it is important to make the appropriate inquiries during the evaluation of the surgical patient.

It is generally recommended that patients discontinue garlic use 7 days prior to surgery to reduce the risk of possible bleeding complications.

Hypertension, nausea and vomiting, prolonged anesthesia effects, and tachycardia are not well-documented sequelae of garlic supplementation.

63
Q

A 32-year-old woman is scheduled to undergo suction lipectomy of the outer thighs and flank areas. One liter of solution composed of Ringer’s lactate solution mixed with 20 mL of 2% lidocaine and one ampule of epinephrine with a concentration of 1:1000 will be used. Which of the following is the amount of lidocaine in the mixture described?

A ) 100 mg

B ) 200 mg

C ) 300 mg

D ) 400 mg

E ) 500 mg

A

The correct response is Option D.

Each liter of solution contains 400 mg of lidocaine with a concentration of 0.04% and 1:1,000,000 epinephrine.

Lidocaine is used more often as the anesthetic agent in the wetting solution. It has a wider range of safety than bupivacaine (Marcaine) and is more easily reversed. Historically, the recommended dose of lidocaine is less than 7 mg/kg. However, this dose does not take into consideration the slow absorption from fat, the persistent vasoconstriction from epinephrine, and the lidocaine removed in the suction lipectomy aspirate, all of which contribute to a reduced risk of systemic toxicity. It is generally accepted that a dose of lidocaine up to 35 mg/kg is safe when injected into the subcutaneous fat with solutions containing epinephrine, although doses up to 50 mg/kg have been utilized.

64
Q

An otherwise healthy 38-year-old woman reports feeling faint when she stands up 1 day after undergoing contouring of the abdomen, back, and thighs. Prior to the contouring procedure, she underwent a Roux-en-Y procedure 2 years ago and lost 183 lb (83 kg). BMI is now 23 kg/m2. Pulse is 120/min, and blood pressure is 90/60 mmHg. Which of the following is the most likely underlying cause of this patient’s condition?

A ) Anemia

B ) Hypothermia

C ) Pain

D ) Skin necrosis

E ) Vitamin deficiency

A

The correct response is Option A.

Many patients who have undergone Roux-en-Y gastric bypass surgery have underlying iron deficiency anemia, vitamin B12 (cobalamin) deficiency, and fat soluble vitamin deficiency. These deficiencies may lead to blood clotting disorder. Significant blood loss may occur during body contouring procedures, which is underestimated, and there is little reserve and potential difficulty with blood clotting from malnutrition. It is important to examine the patient carefully for any signs of hematoma. High output of bloody drainage, possibly followed by no drainage at all from clogging of the drain by blood, should be

Prolonged hypothermia may increase risk of infection or wound healing problems. Pain is more likely to cause hypertension and tachycardia. Skin necrosis is a complication of lower body lift, particularly in patients who smoke or wear tight garments. This complication would not lead to the systemic issues described. Skin necrosis risk can be minimized by limiting undermining.

65
Q

A 44-year-old woman is brought to the emergency department because of the sudden onset of heart palpitations and anxiety 12 hours after undergoing abdominoplasty and liposuction. History includes breast cancer. She uses a transdermal patch for contraception. Height is 5 ft 5 in (165 cm) and weight is 152 lb (69 kg). During the abdominoplasty procedure, 4 L of Ringer’s lactate containing lidocaine 25 mg/L and epinephrine 1 mg/L were infused prior to the first incision, and 4.1 L of aspirate were obtained. Recovery had been uneventful until the current episode. Which of the following is the most appropriate management?

A ) Administration of 0.5 mg alprazolam (Xanax)

B ) CT scan of the chest

C ) Determination of serum lidocaine concentration in the blood

D ) Doppler ultrasonography of the lower extremities

E ) Reassurance and observation

A

The correct response is Option B.

The patient described most likely has an acute pulmonary embolism (PE) and should receive urgent medical attention at the nearest emergency department. Her age, recent diagnosis of breast cancer, and estrogen-based contraceptive use places her at moderate-to-high risk for deep venous thrombosis (DVT) and/or PE following surgery. In addition, the combination of large-volume liposuction and full abdominoplasty as a single procedure further increases her risk for these complications. Heart palpitations and anxiety are common complaints in patients experiencing a PE, as are shortness of breath and hyperventilation, but the diagnosis should not be overlooked in a situation like this, even if it is not a presenting symptom. A CT scan of her chest as dictated in a PE protocol is the appropriate diagnostic study to evaluate for PE and will provide the necessary justification to initiate anticoagulation therapy.

Advising her to go to the nearest emergency department to be monitored for lidocaine toxicity is not appropriate because lidocaine toxicity is not likely to be the correct diagnosis. Although serum lidocaine concentrations peak approximately 12 hours postoperatively, the lidocaine dose that she received was well within the established limits of safe lidocaine administration based on her body weight (7 mg/kg with the use of epinephrine).

Attributing her complaints to a preexisting anxiety disorder could have disastrous consequences in the scenario described. The patient is at a significant risk for a PE based on her medical history and her procedure; therefore, she should receive urgent medical care. Alprazolam (Xanax) is a respiratory depressant that could exacerbate her apparent hypoxia.

Doppler ultrasonography would be useful to evaluate for a lower extremity DVT, but it is not the most appropriate study to diagnose a PE.

Reassurance over the phone and arranging to see the patient in your office early the next morning is inappropriate, as any delay in diagnosis of such a serious complication could be potentially lethal.

66
Q

A plastic surgeon is approached by a guest while attending a fund-raising event. The woman asks whether she should have an abdominoplasty procedure, and they discuss specific details regarding it. The woman subsequently calls the office and makes an appointment. On arrival, she gives a detailed history to the office nurse and is examined by the plastic surgeon. The procedure is scheduled and performed 3 weeks later. At which of the following stages of this scenario was a doctor-patient relationship established?

A ) When the discussion at the fundraiser took place

B ) When the patient made an appointment

C ) When the nurse took a detailed history

D ) When the physician examined the patient

E ) When the procedure was performed

A

The correct response is Option A.

Malpractice is negligence that occurs in the performance of a profession. The four elements of negligence are

  1. Duty owed €”the existence of an obligation
  2. Duty breached €”failure to deliver the obligation
  3. Causation €”the link between breach and harm
  4. Damages €”patient injury

Duty is created when the doctor-patient relationship is established. Simply being a physician does not obligate one to establish that relationship. The doctor-patient relationship is formed when a doctor has professional contact with a patient, but it is not necessary to see the patient physically to form this relationship. Anyone the physician supervises and has authority over, even an office clerk, can form the doctor-patient relationship with the physician. If the plastic surgeon gives specific advice, it establishes a doctor-patient relationship. A physician should never enter into a doctor-patient relationship by accident.

67
Q

A 35-year-old man is scheduled to undergo a medial thigh lift. He underwent bariatric surgery 2 years ago and has lost 120 lb (54 kg). To decrease the risk of seroma formation, care must be taken to preserve the area defined by the inguinal ligament and which of the following additional structures?

A ) Adductor longus muscle and sartorius muscle

B ) Gracilis muscle and adductor magnus muscle

C ) Great saphenous vein and sartorius muscle

D ) Iliopsoas muscle and adductor magnus muscle

A

The correct response is Option A.

The incidence of seroma following medial thigh lift ranges from 4 to 20%, making it one of the most common complications of body contouring. It is critical to have an appreciation of the anatomical borders of the femoral triangle for two reasons. First, the concentration of lymphatics draining the lower extremity is very high within the femoral triangle. Second, direct injury to the inguinal lymphatic system often occurs inconspicuously, and so a preemptive approach should be taken to protect these structures.

The femoral triangle is bordered by the inguinal ligament, adductor longus muscle, and sartorius muscle. The iliopsoas muscle, pectineus muscle, inguinal lymph nodes, and femoral nerve, artery, and vein reside within the femoral triangle, as does the proximal portion of the great saphenous vein.

68
Q

For each patient with excessive abdominal tissue, select the most appropriate surgical procedure (A €“E).

A ) Abdominoplasty

B ) Abdominoplasty and liposuction

C ) Circumferential lower body lift (hemi-body lift)

D ) Liposuction

E ) Panniculectomy

A 49-year-old woman who underwent gastric bypass surgery. She has lost 100 lb, going from a body mass index (BMI) of 45 kg/m2 to 28 kg/m2, and now has excess anterior abdominal and buttock tissue.

A 55-year-old man with a BMI of 40 kg/m2, who has had episodes of severe intertrigo and skin infections from overhanging skin and fat of the anterior abdomen. He has smoked one half of a pack of cigarettes daily for five years.

A

The correct response for Item 1 is Option C and for Item 2 is Option E.

For the massive weight-loss patient, only a circumferential lower body lift adequately addresses excessive tissue of both the anterior and posterior trunk.

For the patient with skin infections, a panniculectomy will address the excessive abdominal pannus, which is producing the symptoms described. In a patient with a history of smoking, there is an increased risk of wound-healing complications and skin necrosis. Other surgical options that involve undermining tissues and raising skin flaps will further jeopardize the vascularity and healing of the overlying skin. Liposuction would address the excess adipose tissue but not the excess skin. A straight panniculectomy without undermining will minimize the risk of wound complications in a patient with a history of smoking. Patients who smoke should be counseled on the importance of smoking cessation at least four weeks before and after elective surgery.

69
Q

A 42-year-old man develops a dehiscence of the abdominal incision six weeks after undergoing a lower body lift. Medical history includes a 100-lb (45-kg) weight loss during the past three years. Which of the following is the most likely cause of the wound-healing problem?

A ) Hematoma

B ) Patient movement

C ) Seroma

D ) Skin necrosis

E ) Wound infection

A

The correct response is Option C.

Body lift procedures after massive weight loss have a complication rate of approximately 50%. The most common complication is wound dehiscence, which occurs in greater than 30% of patients. Wound dehiscence can be characterized as either early (in the immediate postoperative period) or late. Early wound dehiscence may be caused by patient movement, while late wound dehiscence is often due to underlying seroma. Although infection and skin necrosis can occur in the postoperative period and result in wound dehiscence, seroma is much more common.

70
Q

Which of the following is most indicative of the aging male abdomen as compared with the aging female abdomen?

A ) Accumulation of fat peripherally below the iliac crest

B ) Greater skin laxity

C ) Less intra-abdominal accumulation of fat

D ) Thinner skin with tendency toward striation

E ) Wider upper rectus muscle diastasis

A

The correct response is Option E.

Men are more likely to have a wide upper rectus muscle diastasis, and women are more likely to have a lower rectus muscle diastasis. Men €™s skin tends to be thicker and less prone to laxity and striation. The accumulation of flank fat is more central in men and located more peripherally below the iliac crest in women. Men have more intra-abdominal accumulation of fat, although women can accumulate intra-abdominal fat more commonly after menopause.

71
Q

A 34-year-old woman is scheduled to undergo brachioplasty following a 100-lb (45-kg) weight loss. Which of the following is the most likely unfavorable outcome of the procedure in this patient?

A ) Contour deformities

B ) Lymphedema

C ) Neuroma

D ) Seroma

E ) Unattractive scarring

The correct response is Option E.

The most common complication of brachioplasty in postbariatric body contouring is hypertrophic, widened, or unattractive scarring. This multifactorial problem is secondary to problems in geometry (tension and compression), multiple vectors, tissue mismatch, nutrition, and adequate resection. The plastic surgeon must strike a balance between resecting enough tissue to achieve excellent contour and not resecting so much that closure is prevented. Other complications include wound separation, hematoma, seroma, infection, sensory and motor nerve injury, over- or under-resection, fat necrosis, horizontal cutaneous furrows, standing cone deformities, edema (venous), lymphedema, and inappropriate scar placement.

A

The correct response is Option E.

The most common complication of brachioplasty in postbariatric body contouring is hypertrophic, widened, or unattractive scarring. This multifactorial problem is secondary to problems in geometry (tension and compression), multiple vectors, tissue mismatch, nutrition, and adequate resection. The plastic surgeon must strike a balance between resecting enough tissue to achieve excellent contour and not resecting so much that closure is prevented. Other complications include wound separation, hematoma, seroma, infection, sensory and motor nerve injury, over- or under-resection, fat necrosis, horizontal cutaneous furrows, standing cone deformities, edema (venous), lymphedema, and inappropriate scar placement.

72
Q

A 35-year-old woman is scheduled to undergo liposuction in an office-based setting. Height is 5 ft 10 in (178 cm) and weight is 185 lb (84 kg). According to the American Society of Plastic Surgeons Committee on Patient Safety, the recommended maximum total volume of aspirate should be limited to which of the following amounts?

A ) 4000 mL

B ) 5000 mL

C ) 6000 mL

D ) 7000 mL

E ) 8000 mL

A

The correct response is Option B.

In the €œPractice Advisory on Liposuction, € the American Society of Plastic Surgeons Committee on Patient Safety defines large-volume liposuction as more than 5 L of lipoaspirate taken in one operation. Different states may have different regulations regarding liposuction aspirate volume.

Liposuction should be performed in an appropriate facility with qualified staff to monitor postoperative vital signs and urinary output. A postliposuction surgery protocol should be followed to monitor and document any signs of volume overload. These signs include:

increased blood pressure, jugular vein distension, and full bounding pulses

cough, dyspnea, lung crackles, pulmonary edema, and changes in urinary output

The physician should not combine large-volume liposuction with additional procedures because of the risk of severe complications.

73
Q

A 30-year-old woman has numbness of the forearm following a brachioplasty procedure to correct significant skin laxity. Which of the following nerves was most likely injured during the procedure?

A ) Axillary

B ) Intercostal brachial

C ) Medial antebrachial cutaneous

D ) Posterior interosseous

E ) Ulnar

A

The correct response is Option C.

The medial antebrachial cutaneous (MABC) nerve may be located within the resection area of the arm during brachioplasty, leading to regional paresthesia in the upper arm and the anterior proximal forearm. The MABC nerve exits 14 cm proximal to the medial epicondyle and tends to run in close proximity to the intramuscular septum. Patients may be treated with hand therapy and local massage, as well as medications such as gabapentin (Neurontin), if needed. Sensation often improves with the passage of time.

74
Q

A 57-year-old woman is scheduled to undergo a medial thigh lift. Anchoring of the soft tissue of the medial thigh to the Colles fascia is planned. The Colles fascia is continuous with which of the following structures?

A ) Anococcygeal raphe

B ) Anterior rectus sheath

C ) Deep layer of the deep perineal fascia

D ) Posterior urogenital diaphragm

E ) Scarpa €™s fascia of the abdominal wall

A

The correct response is Option E.

Colles fascia was first described in 1811 as a fascial layer that helps to define the perineal-thigh crease. It is described as the deep layer of the superficial perineal fascia (not the deep perineal fascia) and lies deep to the subcutaneous fat of the perineum. Anteriorly, it is continuous with Scarpa €™s fascia of the abdominal wall. It is distinct, unrelated, and not continuous with the anterior rectus sheath. Posteriorly, Colles fascia fuses with the posterior border of the urogenital diaphragm. It does not occur posterior to this structure. The Colles fascia is high in elastin content, giving it a yellow hue which distinguishes it from nearby white muscular fascia.

Lockwood initially described the fascial anchoring technique to be employed at medial crescentic thigh lift at which the inferior thigh flap is anchored to this fascial layer with subdermal sutures to reduce the risk of inferior scar migration, labial spreading, and recurrent thigh ptosis.

75
Q

A 56-year-old woman comes to the office for consultation regarding aesthetic improvement of her abdomen. She says she is unhappy about the excess abdominal fat and skin, an unattractive abdominal contour from a previous surgery, and the size and ptosis of the mons area. A fleur-de-lis abdominoplasty is planned. To achieve optimal aesthetic improvement of the mons area, the lower incision should be made at which of the following locations?

(A) 2.5 cm above the vulvar commissure

(B) 5 cm above the vulvar commissure

(C) Panniculus crease

(D) Pubic hairline

(E) Vulvar commissure

A

The correct response is Option B.

To maximize the lift in the mons area without incurring an abnormal contour, the incision should be placed 5 to 7 cm above the vulvar commissure and within the hairline. If there is excessive fat within the mons area, it can be treated with suction lipectomy or defatting. Undermining and deep fat excision should be avoided because they may cause injury to the sensory nerves and untoward lymphedema. Liposuction to the area may assist in an improved contour. The superficial fascial system of the mons is then fixed to the rectus fascia with care to match the thickness of the upper and lower flaps.

76
Q

A 30-year-old woman comes to the office for consultation regarding a body lift two years after undergoing a gastric bypass procedure. On initial evaluation, the presence of which of the following medications would increase the patient €™s risk for an embolic event during surgery?

(A) Anticholesterol medications

(B) Antidepressants

(C) Diuretics

(D) Oral contraceptives

(E) Thyroid replacement

A

The correct response is Option D.

Patients undergoing bariatric procedures have many of the known risk factors for embolic events, such as long procedure duration, immobilization, and obesity. The risk of deep venous thrombosis, even with prophylaxis, has been reported to be as great as 9%. Oral contraceptives and estrogen replacement therapy are known to increase the rate of embolic problems, and patients should be counseled to stop these medications, if possible, before the procedure. Current recommendations for prophylaxis include proper positioning during surgery, early ambulation, antiembolic stockings, and pneumatic compression stockings. Consideration may also be given to the use of low-molecular-weight or unfractionated heparin before and after surgery.

Antidepressants, anticholesterol medications, thyroid replacement, and diuretics have not been associated with increased risk for pulmonary embolism.

77
Q

A 36-year-old woman is scheduled to undergo a circumferential body lift procedure for correction of skin laxity resulting from a 100-lb weight loss. After the preoperative consultation, a simultaneous procedure to correct the flat and unnatural appearance of the patient €™s buttocks is planned. Which of the following is the most appropriate method of buttock augmentation for this patient?

(A) Autologous gluteal flaps

(B) Autologous fat injections

(C) Silicone gel prostheses

(D) Solid silicone prostheses

The correct response is Option A.

Autologous gluteal flaps provide easily accessible donor tissue that can be used simultaneously in body lift procedures. The dermal fat flaps can be elevated safely to autoaugment the buttocks. This technique adds no more than 45 minutes to the procedure and does not add to the usual complications of body lift procedures in general.

Autologous fat transfer can be used to enhance buttock shape, but unfortunately the buttocks of many of these patients are deflated and do not have adequate donor sites for grafting. Free tissue transfers are not recommended for buttock augmentation. Silicone prostheses, both solid and gel types, are associated with less than optimal outcomes because of palpability and a greater risk of infection and dehiscence.

Augmentation with fat injections from liposuction is possible, but the results are less predictable than the use of vascularized tissue.

The correct response is Option A.

Autologous gluteal flaps provide easily accessible donor tissue that can be used simultaneously in body lift procedures. The dermal fat flaps can be elevated safely to autoaugment the buttocks. This technique adds no more than 45 minutes to the procedure and does not add to the usual complications of body lift procedures in general.

Autologous fat transfer can be used to enhance buttock shape, but unfortunately the buttocks of many of these patients are deflated and do not have adequate donor sites for grafting. Free tissue transfers are not recommended for buttock augmentation. Silicone prostheses, both solid and gel types, are associated with less than optimal outcomes because of palpability and a greater risk of infection and dehiscence.

Augmentation with fat injections from liposuction is possible, but the results are less predictable than the use of vascularized tissue.

A

The correct response is Option A.

Autologous gluteal flaps provide easily accessible donor tissue that can be used simultaneously in body lift procedures. The dermal fat flaps can be elevated safely to autoaugment the buttocks. This technique adds no more than 45 minutes to the procedure and does not add to the usual complications of body lift procedures in general.

Autologous fat transfer can be used to enhance buttock shape, but unfortunately the buttocks of many of these patients are deflated and do not have adequate donor sites for grafting. Free tissue transfers are not recommended for buttock augmentation. Silicone prostheses, both solid and gel types, are associated with less than optimal outcomes because of palpability and a greater risk of infection and dehiscence.

Augmentation with fat injections from liposuction is possible, but the results are less predictable than the use of vascularized tissue.

78
Q

A 45-year-old woman is undergoing abdominoplasty. She does not smoke cigarettes and currently takes estrogen replacement medication. Medical history includes a deep venous thrombosis with no pulmonary embolus 12 years ago. Which of the following is the most appropriate thromboembolism prophylaxis in this patient?

(A) Administration of aspirin

(B) Administration of enoxaparin and application of sequential compression devices

(C) Administration of warfarin pre- and postoperatively

(D) Application of elastic stockings and intravenous administration of heparin

(E) Early ambulation

A

The correct response is Option B.

Deep venous thrombosis (DVT) and pulmonary embolus (PE) can be deadly sequelae of hospitalization, immobilization, and general anesthesia. These risks can be minimized with proper prophylaxis at the time of surgery. This is especially important in plastic surgery: liposuction, abdominoplasty, belt lipectomy, and combined procedures all have a significant incidence of embolic risk.

Patients are generally divided into categories of risk. In 2002, the American Society of Plastic Surgery Task Force issued a practice guideline that divided patients into three levels of risk. Low-risk patients are under 40 years of age and are undergoing minor procedures. Moderate-risk patients are over 40 years of age and are under general anesthesia for more than 30 minutes. Medications such as estrogen cause moderate risk. High-risk patients have additional risk factors such as age over 60 years, long-duration operations, immobilization, malignancies, prior embolic disease, and hypercoagulable states.

Surgeons should have a high index of suspicion for DVT and PE. Patients with shortness of breath and tachypnea should be immediately evaluated for PE. Leg edema, calf tenderness, or palpable cords can be signs of DVT.

Evidence-based guidelines from the 2004 American College of Chest Physicians €™ Conference state that (1) aspirin alone should not be used as prophylaxis for any group; (2) for moderate-risk patients, unfractionated heparin or low-molecular-weight heparin is recommended; (3) for high-risk patients, prophylaxis with heparin three times per day or higher-dose low-molecular-weight heparin is recommended; (4) mechanical prevention should be used in patients who are at high risk for bleeding and as an adjunct to anticoagulation-based prophylaxis in other risk groups.

Each local hospital may also have protocols for the prevention of thromboembolism, which can easily be formatted into the written operative orders.

79
Q

A 54-year-old woman who recently had a 40-lb weight loss is scheduled to undergo a medial thigh lift procedure. Which of the following closure techniques for this procedure is most effective in decreasing the risk of lymphedema of the legs and knees?

(A) Anchoring the inferior skin flap to the superficial layer of the superficial fascia of the perineum

(B) Closing the flap to the recipient areas using the Scarpa fascia

(C) Superficial dissection over the femoral triangle

(D) Suturing the superficial fascial system of the flap to the Colles fascia

(E) Undermining the inferior skin flap deep to the adductor muscle fascia

A

The correct response is Option C.

The superficial fascia of the medial thigh consists of two layers: superficial and deep. The superficial layer is thick, loose, areolar in texture, and contains in its meshes much adipose tissue, the amount of which varies in different patients. The deep layer (Colles fascia) is thin, aponeurotic in structure, and of considerable strength.

In an attempt to limit untoward results, the medial thigh lift was modified to allow anchoring of the inferior skin flap to the tough and inelastic deep layer of the superficial fascia of the perineum. Using the Colles fascia as the central anchor for the medial thigh lift has produced more consistent and long-lasting results, decreasing the risk of problems commonly associated with the classic skin €‘suspension medial thigh lift €”widened scars and labial distortion. In the groin, the depth of the incision is to the fascia lata in the region between the ischial tuberosity and the symphysis pubis. Lymphedema of the knee and leg can result if lymphatic vessels are damaged in the course of a medial thigh lift. The depth of the incision should remain superficial through the fat over the femoral triangle in order to leave lymphatics intact.

80
Q

A 45-year-old woman who has had a 100-lb weight loss since undergoing gastric banding two years ago comes to the office for consultation regarding loose skin on her upper arms. Physical examination shows significant ptosis of the posteromedial aspect of the upper arms and relaxation of the soft tissue. Fascia in which of the following regions is also relaxed in this patient and is appropriate for use as an anchor during brachioplasty to reduce widening and hypertrophy of the scars?

(A) Axillary

(B) Bicipital

(C) Deltoid

(D) Pectoral

(E) Tricipital

A

The correct response is Option A.

An important etiologic mechanism of the aesthetic arm deformity is relaxation of a longitudinal fascial system sling that extends from the clavicle to the soft tissues of the posteromedial aspect of the arm via the clavipectoral and axillary fascia. Reanchoring the soft tissues of the posteromedial aspect of the arm to the axillary fascia with nonabsorbable sutures addresses the relaxation of the axillary fascia sling and forms the basis for the anchor brachioplasty. Similarly to anchoring the soft tissues of the medial aspect of the thigh to the Colles fascia (perineum) in medial thigh lifts, fascial anchoring in brachioplasties provides more predictable results while reducing complications.

81
Q

A 40-year-old woman comes to the office for follow-up examination three weeks after undergoing traditional brachioplasty for correction of skin laxity in the proximal aspect of the upper arms. She has had persistent numbness along the medial aspect of the elbow and forearm, extending to the level of the wrist. Which of the following nerves was most likely injured during brachioplasty in this patient?

(A) Intercostobrachial

(B) Medial antebrachial cutaneous

(C) Medial brachial cutaneous

(D) Musculocutaneous

(E) Posterior brachial cutaneous

A

The correct response is Option B.

Traditional methods of brachioplasty use a medial or posteromedial longitudinal incision to resect excess skin and fat. The medial antebrachial cutaneous nerve branches, together with the basilic vein, exit the deep fascia of the medial arm at the transition between the middle and distal thirds of the arm. Care should be taken at the time of surgery to identify this nerve at this level and maintain more fat on the deep fascia to prevent injury.

The intercostobrachial nerve and the medial brachial cutaneous nerve innervate the medial arm skin. The musculocutaneous nerve terminates in the lateral antebrachial cutaneous nerve, supplying sensation to the lateral forearm. The posterior brachial cutaneous nerve, a branch of the radial nerve, supplies sensation to the posterior arm.

82
Q

A 41-year-old woman comes to the office for consultation regarding irregular skin tone and uneven skin surface on the outer thighs one year after she underwent liposuction of the hips and thighs. On physical examination, the irregularities are radially oriented around a single small scar approximately 1 cm wide. No other scars are visible. The most likely explanation for these findings is use of which of the following during the liposuction procedure?

(A) Deep large cannula suction with cross-tunneling

(B) Fine cannula suction with cross-tunneling

(C) Fine cannula suction without cross-tunneling

(D) Superficial large cannula suction through a single port site

(E) Superficial large cannula suction with cross-tunneling

A

The correct response is Option D.

One of the most common deformities of liposuction is surface irregularity, which can result from several etiologies. A large cannula will create a large furrow that may be visualized if the liposuction is not done evenly. Use of a single port may also lead to irregularities because the suctioning is done from only one angle. Superficial suctioning is also more prone to visible irregularities. The best way to avoid this problem is to use small cannulas in the deep fat, with cross-tunneling from two sites such that the tunnels are at right angles to each other.

83
Q

A 27-year-old woman (weight, 60 kg) undergoes suction lipectomy of the abdomen, flanks, and outer thighs in an outpatient setting. During the procedure, 3 L of tumescent solution containing 0.1% lidocaine and 1:1,000,000 epinephrine in normal saline are infused and 3 L of aspirate are removed using a 20-cm-long, 4-mm-wide cannula. The patient is discharged in good condition two hours after completion of the procedure. She is found dead in her bed the next morning. Which of the following is the most likely cause of death?

(A) Epinephrine toxicity

(B) Lidocaine toxicity

(C) Perforation of the abdominal wall and colon

(D) Perforation of the diaphragm and tension pneumothorax

(E) Pulmonary embolism

A

The correct response is Option E.

Pulmonary embolization with fat or thrombus is the most common cause of death from suction lipectomy. One survey of 917 aesthetic plastic surgeons showed the leading cause of death from suction lipectomy to be thromboembolism (23%), followed by perforation of the abdomen/viscera (15%), anesthesia/sedation/medication (10%), fat embolism (9%), and unknown or confidential causes (29%).

84
Q

A 45-year-old woman comes to the office for consultation regarding severe skin laxity of the lower abdomen and medial aspect of the thighs two years after she underwent gastric bypass surgery. Weight loss during the first 12 months postoperatively was 120 lb. The patient €™s weight has been steady at 140 lb for the past eight months. Which of the following incisional approaches is most appropriate for contour of the thighs in this patient?

(A) Anterior oblique

(B) Crescentic groin

(C) Medial longitudinal

(D) Posterior oblique

(E) Transverse

A

The correct response is Option C.

In a patient with severe medial thigh skin redundancy after massive weight loss, a medial longitudinal approach is currently preferred because it removes a maximal amount of skin redundancy along the entire length of the thigh while concealing the incision in the medial thigh skin. A crescentic groin incision does attempt to conceal the scar in the groin but can distort the vulvar structures when placed under tension, and is less effective when medial thigh laxity is severe. An anterior oblique or posterior oblique incision is not preferred because of the conspicuous location of the resulting scars. A transverse incision is inadequate to remove enough skin and results in a poor cosmetic outcome.

85
Q

A 50-year-old man who underwent uncomplicated full abdominoplasty with suction lipectomy two weeks ago comes to the emergency department because he has had painless swelling of the right lower extremity for the past 24 hours. He currently smokes one pack of cigarettes daily. Medical history includes no serious disease conditions. Vital signs are stable and within normal limits. Physical examination shows 7-cm enlargement of the circumference of the right leg with a positive Homans sign. The chest is clear to auscultation, and the patient is not in acute distress. Which of the following diagnostic studies is the most appropriate initial step?

(A) Doppler ultrasonography of the lower extremities

(B) Lymphangiography

(C) Spiral CT of the chest

(D) Venography of the lower extremities

(E) Ventilation-perfusion (VQ) scan

A

The correct response is Option A.

Among plastic surgery procedures, abdominoplasty has one of the highest rates of deep vein thrombosis (DVT) and pulmonary embolism. It is essential to know the proper course of management of this major cause of postoperative morbidity and mortality. Clinical workup may vary with the severity of the symptoms and physical examination of the patient, and not all clinical investigations may be available to the surgeon. However, a clear path of investigation and treatment should be developed.

A Doppler ultrasonography is the most appropriate initial test for DVT because it is quick, noninvasive, and readily available in most centers.

Venography of the lower extremities is also a diagnostic test, but it is invasive and has increased risks.

Lymphangiography is a diagnostic test for lymphedema, not for DVT.

If pulmonary embolism is suspected, a spiral CT of the chest or a VQ scan of the lungs is indicated.

86
Q

A 37-year-old woman is undergoing infiltration of wetting solution in preparation for liposuction of the trochanteric region. As the infiltration progresses, supraventricular tachycardia is noted on the electrocardiography monitor. Blood pressure remains within normal limits. Which of the following is the most appropriate next step?

(A) Administer procainamide 100 mg slowly over 5 minutes to a maximum dose of 1000 mg and perform 12-lead electrocardiography

(B) Administer protamine 25 mg by intravenous infusion over 10 minutes and perform 12-lead electrocardiography

(C) Continue the procedure as planned after obtaining chest radiographs

(D) Discontinue the infiltration, check the composition of the infiltrate, and assess for possible accidental intravascular injection

(E) Discontinue the infiltration until the tachycardia resolves and immediately cardiovert

A

The correct response is Option D.

The infiltration of large amounts of lidocaine and epinephrine is not without risk. Medication errors are missed because the surgeon fails to check that the operating room personnel have properly constituted the wetting solution, with severe consequences. Wetting solution preparation is often relegated to junior personnel who are unaware of the importance of correct formulation. Intravascular injections can also contribute to lidocaine or epinephrine complications and toxicity.

Supraventricular tachycardia is a normal heart response to adrenaline, but it nevertheless should be assessed by the surgeon and anesthesiologist with a good electrocardiogram. The infiltration should be discontinued. The vital signs of the patient described will determine further treatment.

Beta blockers, calcium channel blockers, procainamide, and other medications may be used if the patient €™s rhythm does not normalize. Protamine is used to reverse heparin and is not used for arrhythmia. Chest radiographs will not diagnose the etiology or the arrhythmia. Cardioversion is not used if the patient €™s blood pressure is stable.

87
Q

One year after undergoing gastric bypass surgery, a 36-year-old woman comes to the office for consultation regarding cosmetic improvement of the lower torso. She has lost 120 lb since the procedure was performed, and her weight has been stable for the past three months. She does not smoke cigarettes. Height is 5 ft 4 in and weight is 175 lb. Physical examination shows circumferential laxity of the truncal skin with lipodystrophy of the hips and rectus diastasis as well as an upper midline incision. No hernias are noted. A lower body lift is scheduled. Which of the following is the most likely complication of this procedure in this patient?

(A) Infection

(B) Postoperative hemorrhage

(C) Pulmonary embolism

(D) Seroma

(E) Wound dehiscence

A

The correct response is Option D.

The most common complication of a lower body lift procedure is seroma. In a large retrospective series, seromas occurred in 37.5% of all patients; wound dehiscence (12%), infection (8%), and pulmonary embolism (6%) also occurred frequently after this type of surgery. Other complications such as psychiatric difficulties, unscheduled hospitalizations, skin necrosis, and postoperative hematomas may be common but are not reported as often. Complications seem to be associated with higher preoperative body mass index (weight) and history of smoking.

88
Q

A 50-year-old woman comes to the office for consultation regarding body contouring one year after undergoing a gastric bypass procedure. She has lost 160 lb since the procedure was performed. Preoperative photographs are shown above. Which of the following is the most appropriate surgical procedure for improvement of the thigh and buttock regions in this patient?
(A) Direct excision of gluteal skin folds
(B) Suspension of a dermal fat flap
(C) Augmentation of the gluteal region
(D) Liposuction
(E) Lower body lift

A

The correct response is Option E.

In this patient, the best option for improvement of the buttock and thigh regions is a lower body lift. This procedure is indicated in patients with relaxation and resultant ptosis of the buttock and thigh tissues. In addition to moderate or more severe skin laxity, the potential for laxity to develop after liposuction is an indication for an excisional lifting procedure over liposuction alone. In this patient, placing the incision at the level used in the lower body lift will create a better hip contour as well as lift the descended thigh and buttock tissue. This can be combined with thigh liposuction to address any thigh lipodystrophy.

Direct excision of the gluteal fold can result in flattening of the buttocks, asymmetry, and hypertrophic scarring. Suspension of a dermal-fat flap is performed using a gluteal fold incision. Flattening of the buttocks and loss of gluteal fold definition, as well as hypertrophic scarring, can also result from this technique. Gluteal implants have the associated risks of implant extrusion, infection, and malposition. Furthermore, these three techniques do not address the primary pathology of the descent of tissue and do not restore the tissues to a more anatomic position. Liposuction can be used in the patient with good skin quality where adequate retraction is likely. However, if there is potential for skin laxity, liposuction may result in a worsening of the deformity as the tissue is deflated. Liposuction also carries a risk of worsening buttock ptosis, loss of gluteal fold definition, and contour irregularities. Liposuction can be used as an adjunct to excisional techniques.

89
Q

A 50-year-old man who weighs 155.5 lb (70 kg) is scheduled to undergo liposuction of the abdomen, flanks, and chest with administration of epinephrine and lidocaine for tumescent anesthesia. Which of the following amounts of lidocaine is the maximum for this patient?
(A) 7 mg
(B) 35 mg
(C) 150 mg
(D) 490 mg
(E) 2450 mg

A

The correct response is Option E.

For subcutaneous infiltration with 1% lidocaine with 1:100,000 epinephrine, 7 mg/kg of lidocaine is generally recommended. It has been shown, however, that the use of “tumescent fluid” with 1:1,000,000 epinephrine can be safely given with lidocaine doses as high as 35 mg/kg. Lidocaine toxicity can have an excitatory effect on the nervous system, such as tingling, numbness, mental status changes, and, eventually, seizures. Subsequent manifestations include CNS depression with cessation of convulsions, unconsciousness, and respiratory depression or arrest. In the heart, this mechanism depresses Vmax (i.e., the rate of depolarization during phase 0 of the cardiac action potential) and might lead to reentrant arrhythmias. Additionally, conduction through the sinus and atrioventricular (AV) nodes is suppressed. Acceleration of the ventricular rate has been reported in patients with atrial arrhythmias. Lidocaine also might elevate fibrillation thresholds. A negative inotropic effect on the myocardium and direct peripheral vasodilation might occur, which can produce hypotension. Lidocaine commonly is associated with sinus tachycardia, whereas bupivacaine has been known to cause ventricular tachycardia and fibrillation. Widening of the PR interval, increased QRS duration, sinus tachycardia, sinus arrest, and partial or complete AV dissociation can be seen.

90
Q

When performing liposuction using a tumescent technique, the ratio of infiltrate to aspirate is closest to which of the following?
(A) 1:1
(B) 1:2
(C) 1:3
(D) 2:1
(E) 3:1

A

The correct response is Option E.

Tumescent suction lipectomy calls for the largest volume of infiltrate, using up to 3 or 4 ml of infiltrate for each 1 ml of planned aspirate. With this technique, blood loss is approximately 1% of the aspirate volume.

The first method of suction lipectomy, the dry technique, was performed under general anesthesia without infiltration of subcutaneous solutions before insertion of the suction lipectomy cannula. This technique commonly caused substantial swelling and discoloration and blood loss of 20% to 45% in the suction aspirate. Except in limited applications, this approach has been abandoned.

In the wet technique, 200 to 300 ml of infiltrate or wetting solution, with or without additives, is injected into the operative field before insertion of the suction lipectomy cannula. Small doses of the vasoconstrictor epinephrine are added to the infiltrate, which decreases blood loss to 4% to 30% of the aspirate.

The superwet technique uses a larger volume of subcutaneous infiltrate, obtaining a 1-to-1 ratio of infiltrate to aspirate. The infiltrate consists of saline or Ringer’s lactate solution, epinephrine, and in some cases, lidocaine. With this technique, blood loss is comparable with that of the tumescent technique, at approximately 1%.

91
Q

A 36-year-old woman who underwent panniculectomy three weeks ago has infection of the surgical wound requiring open drainage and packing with daily dressing changes. Medical history includes Roux-en-Y gastric bypass surgery 20 months ago. Physical examination shows a 35 x 12-cm open wound that has not decreased in size since completion of the procedure. No significant formation of granulation tissue is shown, but the wound is otherwise clean; no signs of fat deposits or necrosis are noted. Which of the following interventions is the most appropriate management?
(A) Administration of an antibiotic
(B) Nutritional supplementation
(C) Hyperbaric oxygen therapy
(D) Primary closure
(E) Surgical debridement

A

The correct response is Option B.

Roux-en-Y gastric bypass (RYGB) is the most commonly performed bariatric procedure and combines elements of restrictive and malabsorptive procedures. The size of the stomach is reduced by stapling, and a variable segment of small bowel is bypassed using a Roux limb. Major elective surgery will require an increased calorie and protein consumption of approximately 25%.

Most bariatric patients who have stabilized in their weight loss should be able to meet these needs. However, burns, trauma, infection, and large open wounds can substantially increase these nutritional requirements to the point at which they cannot be met through oral intake. In addition to increased metabolism resulting from these pathologic processes, a substantial amount of protein can be lost from a large wound. Exudate from an open wound may contain up to 44 mg/ml protein and result in losses that the bariatric patient cannot match. In this patient, the large open wound has failed to show expected progress in healing. Aggressive nutritional evaluation and supplementation should be performed. If oral intake with protein supplements is not adequate, endoscopic placement of an enteral feeding tube or total parenteral nutrition may be necessary. Hyperbaric oxygen therapy is unlikely to improve healing in this case. Surgical debridement is not necessary because the wound is free of necrosis. Wound biopsy would be appropriate for a chronic non-healing wound to exclude malignancy.

92
Q

With the tumescent liposuction technique, which of the following percentages of the aspirate is blood?
(A) 1%
(B) 4%
(C) 8%
(D) 12%
(E) 15%

A

The correct response is Option A.

With tumescent liposuction, blood makes up 1% of the aspirate. This technique produces the greatest degree of hemostasis. This technique consists of infiltration to skin turgor, which represents approximately 2 to 3 ml of infiltrate to 1 ml of aspirate.

Superwet technique consists of 1 ml of infiltrate for each 1 ml of aspirate to be removed. This technique also has an estimated 1% blood loss in the aspirate.

Wet technique consists of infiltration of 200 to 300 ml per area and is not based on the amount to be aspirated. With this technique, 4% to 30% of the aspirate is blood.

In the dry technique, which does not involve any infiltration of the tissue, blood has been estimated to comprise 20% to 45% of the aspirate.

93
Q

A healthy 60-year-old woman undergoes suction lipectomy of the lateral thighs during which a total volume of 2 L is aspirated. During the procedure, 1.5 L of infiltrate and 1.5 L of crystalloid are administered intravenously. Which of the following is the most appropriate additional intervention for fluid management?

(A) Administer a 1-L bolus of crystalloid
(B) Administer a 3-L bolus of crystalloid
(C) Administer a diuretic
(D) No further hydration

A

The correct response is Option A.

Hydration during suction lipectomy is very important to prevent complications. Too little fluid causes hypotension and too much fluid can cause pulmonary edema. As larger amounts of infiltrate are used with the tumescent technique, it is difficult to know how much intravenous fluid to give to the patient.

One study concludes that the combination of infiltrate and intravenous fluids (1.5 L and 1.5 L in this case) should add up to twice the aspirate removed (2 _ 2 L). Therefore, this patient should receive 1 L of fluid in the recovery area. Careful monitoring of blood pressure, tissue turgor, and urine output will confirm that the patient is euvolemic.

Patients should not be discharged before adequate fluids are given to maintain normal urine output. Large-volume liposuctions may require overnight monitoring. Administration of a diuretic is inappropriate because the patient is not overloaded with fluid. A 3-L bolus may overhydrate the patient and lead to pulmonary edema.

94
Q

The incidence of complications is highest following abdominoplasty when the procedure is combined with suction lipectomy of which of the following areas?

(A) Central infraumbilical
(B) Epigastric
(C) Flank
(D) Lateral thigh
(E) Medial thigh

A

The correct response is Option A.

Suction lipectomy of the central portion of the abdominoplasty flap can lead to loss of skin in the central and inferior portions of the flap and should be avoided. A critical step in avoiding skin loss is leaving intact the subcutaneous layer of fat between the skin and the fascia of Scarpa. Only very limited, cautious suction lipectomy should be attempted in the central portion of the abdominoplasty flap and should be limited to globular fat deep to the fascia of Scarpa. The lateral portions of the abdominoplasty flap can be suctioned deep to the fascia of Scarpa. The epigastrium may also be suctioned carefully without complications. The hips and thighs can be suctioned aggressively or defatted directly without concern for skin loss in the abdominoplasty flap.

95
Q

A healthy 30-year-old man is scheduled to undergo suction lipectomy of the trunk using the superwet technique. When injected into subcutaneous fat with solutions containing epinephrine, which of the following is the maximum recommended dose of lidocaine?

(A) 7 mg/kg
(B) 14 mg/kg
(C) 21 mg/kg
(D) 28 mg/kg
(E) 35 mg/kg

A

The correct response is Option E.

The maximum recommended dose of lidocaine is 35 mg/kg when injected into subcutaneous fat with solutions containing epinephrine. Doses of up to 50 mg/kg have been used, but are not generally recommended because of toxicity concerns.

Various individual anesthetic agents and anesthetic combinations are appropriate for suction lipectomy, depending on the patient=s health, the estimated volume of aspirate to be removed, and the postoperative discharge plan. For suction lipectomy, anesthetic agents are added to the wetting solution to provide preemptive and prolonged postoperative local analgesia. In smaller-volume suction lipectomy cases, anesthetic infiltration solutions alone may provide adequate pain relief. In larger-volume suction lipectomy cases, the superwet and tumescent solutions are often used for sedation, general, or epidural anesthesia to ensure adequate patient comfort.

Lidocaine with epinephrine is used most often as the anesthetic agent in the wetting solution. Historically, the recommended dose of lidocaine was less than 7 mg/kg. However, this dose did not take into consideration the slow absorption of lidocaine from fat, the persistent vasoconstriction caused by epinephrine, and the lidocaine removed in the suction lipectomy aspirate. All these factors decrease the risk of systemic toxicity from lidocaine and allow higher doses to be used safely.

96
Q

A 45-year-old man who has achieved substantial weight loss from massive obesity is scheduled to undergo belt lipectomy for circumferential truncal excess. Which of the following is the most likely postoperative complication?

(A) Deep venous thrombosis
(B) Dehiscence
(C) Infection
(D) Seroma
(E) Skin necrosis

A

The correct response is Option D.

Compared with traditional abdominoplasty, belt lipectomy (which combines abdominoplasty with circumferential excision of skin and fat) can provide more optimal contouring for patients with circumferential truncal excess because it addresses the trunk as a unit. Patients who have had a substantial weight loss commonly are candidates for this procedure. After belt lipectomy, care must be undertaken to monitor for and treat complications. Seroma management is a significant part of postoperative care for up to one third of patients who undergo belt lipectomy, and patients should be made aware of the significant morbidity associated with the procedure.

The other complications listed affect less than 10% of patients. Procedures that increase intraabdominal pressure, such as hernia repair, increase the risk of deep venous thrombosis and pulmonary emboli. Deep venous thrombosis and pulmonary emboli are always possible with long-term anesthesia and immobilization, but these are fortunately rare. Wound dehiscence is a concern in belt lipectomy; when the patient flexes at the waist to relieve anterior tension, the back incision is strained and vice versa. Infection rates are less than 5%. Factors that may increase the risk of skin necrosis involve interruption of the lateral intercostal blood supply by lateral skin resection, lack of vascularization across the midline of the anterior abdominal flap, and excess tension on the abdominal flap.

97
Q

During a medial thigh lift procedure, the incision over the femoral triangle is dissected more superficially to avoid injury to which of the following?

(A) Femoral artery
(B) Femoral nerve
(C) Femoral vein
(D) Inguinal ligament
(E) Lymphatic plexus

A

The correct response is Option E.

The lymphatics of the lower extremity travel to the superficial inguinal nodes and deep inguinal nodes. The deep inguinal nodes are situated just medial to the femoral vein and receive lymphatics that accompany the femoral vessels, lymph vessels from the genitalia, and efferents from the superficial nodes. The superficial nodes receive vessels from the gluteal region, infraumbilical abdominal wall, perianal region, and external genitalia as well as most of the superficial lymph vessels of the lower limb. Because superficial lymph vessels are small, they are prone to injury, which can lead to wound-healing problems such as lymphoceles and seromas as well as problems associated with recurrent peripheral edema. The femoral vein, artery, and nerve run deeper, beneath the inguinal ligament at the level of the incision, significantly deeper than the fragile lymphatics of the superficial inguinal nodes.

98
Q

When performing suction lipectomy using the superwet technique, the amount of blood loss in the suction aspirate is closest to which of the following?

(A) 0%
(B) 10%
(C) 20%
(D) 30%
(E) 40%

A

The correct response is Option A.

With the superwet technique, blood loss is approximately 1% of the suction aspirate. This technique uses a 1:1 ratio of subcutaneous infiltrate to aspirate. The infiltrate consists of saline or Ringer’s lactate solution, epinephrine, and in some cases lidocaine.

The first method of suction lipectomy, the dry technique, was associated with blood loss of 20% to 45% in the suction aspirate as well as substantial swelling and discoloration. It was performed under general anesthesia without infiltration of subcutaneous solutions before insertion of the suction lipectomy cannula. Except in limited applications, this approach has been abandoned.

The wet technique is associated with blood loss of 4% to 30% of the aspirate. In this technique, 200 to 300 mL of infiltrate or wetting solution, with or without additives, is injected into the operative field before insertion of the suction lipectomy cannula. Small doses of the vasoconstrictor epinephrine are added to the infiltrate.

Like the superwet technique, tumescent suction lipectomy is associated with blood loss of approximately 1% in the suction aspirate. However, it uses more infiltrate, up to 3 or 4 mL of infiltrate for each planned milliliter of aspirate.

99
Q

Which of the following structures is LEAST likely to be injured during brachioplasty?

(A) Basilic vein
(B) Cephalic vein
(C) Intercostobrachial nerve
(D) Medial cutaneous nerve of the forearm

A

Because the cephalic vein runs anterior and superior to the dissection planes used in brachioplasty, it is not likely to be injured during this procedure. The basilic vein, intercostobrachial nerve, and medial cutaneous nerve of the forearm are aligned slightly medially and posteriorly along the arm in the area of dissection for a standard brachioplasty. Therefore, they are susceptible to injury during the procedure.

100
Q

During suction lipectomy using tumescent anesthesia, total blood loss is expected to be what percentage of the total aspirate?

(A) 1%
(B) 10%
(C) 20%
(D) 30%
(E) 50%

A

The correct response is Option A.

The tumescent technique of suction lipectomy uses a dilute solution of lidocaine and epinephrine as infiltrate in the area that is to be suctioned. With this technique, 2 to 3 mL of infiltrate are used for every 1 mL of fluid that is to be aspirated. Blood loss is significantly decreased with this technique; many studies have shown blood loss of only 1% with tumescent anesthesia.

The dry technique of anesthesia is associated with the greatest blood loss, as 20% to 45% of the aspirate consists of blood. The wet technique infiltrates 300 mL of fluid per region and results in aspirate that is comprised of 25% blood.

The superwet technique has a blood-to-aspirate ratio of 3 to 10% and uses an injected volume of 1 mL for every 1 mL of aspirate.

101
Q

A 32-year-old woman who smokes two packs of cigarettes daily wishes to undergo full abdominoplasty to correct excess skin and fat in the lower abdomen resulting from two pregnancies. The risk for development of complications in this patient is closest to

(A) 10%
(B) 30%
(C) 50%
(D) 70%
(E) 90%

A

The correct response is Option C.

Because conventional abdominoplasty procedures involve significant undermining, there is increased potential for complications, such as skin necrosis, infection, wound dehiscence, seroma, and hematoma. In addition, plastic surgery procedures are associated with a high incidence of adverse effects when performed in patients who smoke. These complications are related to the release of nicotine and carbon monoxide during smoking, which decrease blood flow and oxygen delivery and induces thrombogenesis. There is dysfunction of leukocytes, macrophages, and fibroblasts, leading to impaired wound healing.

One recent study of 132 patients who underwent abdominoplasty reported a complication rate of 48% in smokers, compared with a complication rate of 15% in those patients who did not smoke. Another study of 199 abdominoplasty patients reported a complication rate of 52% in smokers and 24% in nonsmokers.

Patients should be advised to discontinue smoking four to eight weeks before the surgical procedure and for an additional four weeks after surgery. Serum levels of nicotine have been shown to return to normal when patients abstain from smoking for eight weeks. In addition, studies of patients who underwent flap reconstruction showed that complications were decreased significantly when smoking was discontinued a minimum of four weeks before surgery.

102
Q

Compared with traditional suction lipectomy, which of the following is more likely to occur with ultrasonic-assisted suction lipectomy?

(A) Contour irregularity
(B) Major blood loss
(C) Nerve injury
(D) Pulmonary edema
(E) Thermal burns

A

The correct response is Option E.

Ultrasonic-assisted suction lipectomy (UAL) is currently performed as a complement to, not a replacement for, traditional suction lipectomy. This technique involves the transmission of ultrasonic energy via a transducer to cavitate adipocytes and emulsify the liquefied fat, making aspiration of fat easier. UAL is advantageous in removing fat from difficult or fibrous body areas, such as the epigastrium and upper abdomen. It is also purported to stimulate skin retraction after superficial treatment.

Many complications that occur with UAL are similar to those seen with traditional lipectomy procedures, including contour irregularities, perforations, paresthesia and hypoesthesia resulting from nerve injury, seroma formation, and adverse cardiopulmonary effects. In addition, UAL has been shown to elicit greater tissue damage and bleeding than traditional techniques; however, the decrease in hemoglobin level postoperatively is similar with either technique.
UAL is associated with an increased incidence of internal and external cutaneous burns resulting from the intense heat generated by sound waves; this complication does not occur with traditional suction lipectomy. In addition, there are concerns regarding the unknown long-term effects of sound waves and exposure to high temperatures, bond dissociation within molecules, production of free radicals within the body, and other factors.

103
Q

Suction lipectomy is a viable means of breast reduction because the percentage of the female breast that is comprised of fat is closest to

(A) 10%
(B) 25%
(C) 50%
(D) 75%
(E) 90%

A

The correct response is Option C.

Suction lipectomy can be performed to remove fatty tissue in the breast without disturbing parenchymal tissue. It is an attractive option for breast reduction in women who would like to avoid the scarring associated with traditional surgical techniques. According to the results of one recent study that involved a significant population of overweight women, the mean percentage of fat in the breast was 61%. Another study of women predominantly of normal body weight showed a mean percentage of fat of 48%. In addition, these studies showed that younger patients have significant amounts of fat in the breast, even though the percentage of fat was shown to increase with age. Body mass index had more influence on the percentage of breast fat than age, and the amount of fat in the breast could not be accurately assessed on physical examination.

One clinical study reported that breast reduction via suction lipectomy alone produced the best results in women who had well-located or slightly ptotic nipples. Benefits of this technique include the avoidance of scarring and compromise to the blood supply and nerves to the nipple. However, many surgeons remained concerned about the effectiveness of suction lipectomy alone in patients with very large breasts, as well as the lack of skin excision, the potential for induction of microcalcifications, and the viability of pathologic examination of removed fat.

104
Q

A 35-year-old woman who weighs 80 kg (176 lb) is undergoing large volume suction lipectomy with expected removal of 5 L of aspirate. In this patient, the total dose of lidocaine should NOT exceed how many milligrams?

(A) 300
(B) 400
(C) 1600
(D) 2800
(E) 4800

A

The correct response is Option E.

The maximum safe dose of lidocaine is controversial. Some standard anesthesia texts list a maximum safe dose of 300 mg. In contrast, other texts report of maximum dose of 5 mg/kg or 7 mg/kg with the addition of epinephrine. However, more recent studies, from patients undergoing suction lipectomy with tumescent anesthesia, have reported the use of higher doses of lidocaine without complication. Several large studies have reported a maximum safe dose of lidocaine of 35 mg/kg, and some published dermatologic studies have cited the maximum safe dose to be as high as 55 mg/kg.

Although recent articles on fatalities related to suction lipectomy have attributed most patient deaths to pulmonary emboli, there has been speculation regarding the role of lidocaine toxicity, because lidocaine levels are typically not obtained at autopsy. Other recent articles suggest that a portion of the infused lidocaine is subsequently suctioned out, and that the use of general anesthesia versus local anesthesia may affect patient mortality rates.

Of the levels listed, 4800 mg is 60 mg/kg in an 80-kg patient, and is higher than the reported safety standards.

105
Q

Suction lipectomy is an effective procedure for management for each of the following conditions EXCEPT

(A) axillary hyperhidrosis
(B) HIV-associated lipodystrophy
(C) liposarcoma
(D) lymphedema
(E) Madelung’s disease

A

The correct response is Option C.

Suction lipectomy is recommended for many conditions because it effectively removes tissue while limiting incisions. Indications for suction lipectomy include axillary hyperhidrosis, in which apocrine and eccrine glands are removed from the axilla using a superficial technique. This results in reduced sweating with a minimal amount of scarring. Although repeat procedures are necessary in approximately 30% of patients, suction lipectomy produces longer lasting effects than treatment with botulinum toxin (Botox).

In patients with HIV infection, administration of protease inhibitor agents may cause abnormal redistribution of fat. Affected patients may develop adiposity in the abdominal and mandibular regions and atrophy in other areas, such as the nasolabial fold. Suction lipectomy is appropriate to remove the excess fat.

Madelung’s disease, or benign symmetric lipomatosis, is a disorder of unknown cause that is characterized by diffuse growth of nonencapsulated lipomas, especially in the neck, shoulders, and posterior trunk. Suction lipectomy can increase range of motion and provide good cosmesis.

Patients with both congenital and acquired lymphedema can benefit from suction lipectomy to decrease tissue thickness. Although its effects are temporary, it provides significant relief and increases function. Other measures, such as massage and use of compressive garments, can be performed in combination with the lipectomy procedure.

Suction lipectomy is not recommended for removal of malignant tumors, such as liposarcoma, because this method of resection is often incomplete. In addition, such tumors may be seeded and histopathologic examination of the tumor specimen may be compromised.

106
Q

Which of the following subcutaneous infiltration techniques used in a patient undergoing suction lipectomy has an infiltrate-to-aspirate ratio of 1:1?

(A) Dry
(B) Superwet
(C) Tumescent
(D) Wet

A

The correct response is Option B.

The superwet technique was first used for subcutaneous infiltration in the late 1980s. This technique involves the injection of a diluted solution of anesthesia and vasopressors that is of equal volume to the estimated volume of fat removed, or an infiltrate-to-aspirate ratio of 1:1. Blood loss in this technique has been shown to vary from 1% to 4% of the aspirate.

In the dry technique, local anesthetics and epinephrine are not injected.

The tumescent technique involves the injection of 2 to 3 mL of wetting solution for every 1 mL of aspirate. One study of 112 patients who had undergone suction lipectomy reported injection, on average, of 4,600 mL of anesthetic solution and an average removal of 2,657 mL of aspirate. Pumps were used to infiltrate the solution at a rate of 50 to 200 mL/min, depending on the affected region of the body and tolerance of the patient. Blood loss of less 1% was noted in these patients, and virtually all procedures were performed using local anesthesia.

With the wet technique, 100 to 300 mL of fluid is injected into each treatment area regardless of the amount of aspirate removed. However, this technique has been shown to result in extensive bruising when performed using general anesthesia, and blood loss has been shown to be as high as 25%.

107
Q

A 58-year-old woman who recently lost 100 lb desires correction of excess skin and fat in the upper arm but does not want unsightly scars. Which of the following is the most appropriate management?

(A) Circumferential excision lipectomy of the upper arm
(B) Excision lipectomy using a medial approach
(C) Excision lipectomy using a posterolateral approach
(D) Suction-assisted lipectomy

A

The correct response is Option B.

In this patient who desires correction of excess skin and fat in the upper arms, the most appropriate management is excision lipectomy using a medial approach. This procedure effectively removes the skin and fat, leaves only a medial scar, and is appropriate for patients who have lost an excessive amount of weight. Most surgeons recommend an elliptical excision, which results in a scar that extends from the medial epicondyle to the axillary dome. The scar is not visible with the arms in adduction.

Circumferential excision lipectomy provides only limited contour correction and results in unsightly scars. Excision lipectomy via a posterolateral approach leaves a visible scar. Suction-assisted lipectomy is preferred in younger patients who have excess fat without excess skin.

108
Q

A 36-year-old woman has numbness of the anterolateral right thigh one month after undergoing abdominoplasty. The most likely cause is injury to which of the following nerves?

(A) Genitofemoral
(B) Lateral cutaneous
(C) Iliohypogastric
(D) Ilioinguinal
(E) Obturator

A

The correct response is Option B.

In this 36-year-old woman who has numbness of the anterolateral right thigh one month after undergoing blepharoplasty, the lateral cutaneous nerve of the thigh is most likely injured. This nerve arises from L2-3 and passes through the inguinal ligament approximately 1 cm medial to the anterosuperior iliac spine. It then passes superficially to the sartorius muscle and divides into anterior and posterior branches in the thigh. The anterior branch becomes superficial about 10 cm below the anterosuperior iliac spine, supplying sensation to the skin of the anterior and lateral thigh. Injury is likely to result in numbness and dysesthesia in this region.

According to the results of a recent study of 101 abdominoplasty patients, 10% had symptoms consistent with injury to the lateral cutaneous nerve. Other studies have reported rates of lateral cutaneous nerve injury following abdominoplasty ranging from 10% to 32%. It is important that the surgeon use extreme caution when performing lateral dissection during this procedure.

The genitofemoral nerve originates from L1-2 and inserts into the abdomen at a variable distance above the inguinal ligament. This nerve divides into genital and femoral branches; the genital branch supplies sensation to the skin of the scrotum, mons pubis, or labia; the femoral branch supplies sensation to the skin over the upper part of the femoral triangle.

The iliohypogastric nerve arises from L1 and divides into lateral and anterior branches. The lateral branch provides sensation to the skin of the lateral part of the buttocks, and the anterior branch supplies sensation to the skin of the abdomen above the pubis.

The ilioinguinal nerve originates from L1 and passes through the superficial inguinal ring to supply sensation to the skin of the superomedial portion of the thigh and the scrotum or mons pubis.

The obturator nerve arises from L2-4 and courses through the lower pelvis with the obturator vessels. It then enters the thigh to provide sensation to the skin of the medial and lower thigh.

109
Q

A 42-year-old woman who has excess skin and subcutaneous tissue of the lower buttocks is scheduled to undergo excisional lipectomy with the incisions parallel to the gluteal fold. Which of the following is the most likely adverse effect?

(A) Dimpling of the buttocks
(B) Fat necrosis
(C) Flattening of the gluteal fold
(D) Painful scarring
(E) Widening of the gluteal cleft

A

The correct response is Option C.

Adverse effects reported with transverse excision lipectomy include flattening and asymmetry of the buttocks and hypertrophic scarring. Dimpling of the buttocks is more commonly associated with suction lipectomy in the region overlying the gluteal muscles because of the large amount of fibrous septa between the fascia and skin. Widening of the gluteal cleft is uncommon because the incision is made parallel to the gluteal fold.

Fat necrosis does not generally occur in the buttocks because of the good vascularity in this region. Long-term painful scarring is also rare.

110
Q

When performing a transverse thigh/buttock lift, which of the following operative techniques has been shown to decrease the frequency of complications, including widening and inferior migration of scars, traction deformity of the vulva, and early recurrence of thigh ptosis?

(A) Direct undermining of the distal flap
(B) Performing suction lipectomy in conjunction with transverse thigh/buttock lift
(C) Suspension of the superficial fascial system
(D) Use of an anterior medial skin resection pattern

A

The correct response is Option C.

Suspension of the superficial fascial system of the inferior skin flap has decreased the incidence of unfavorable scars, vulvar traction, and ptosis deformities previously associated with the transverse thigh/buttock lift. Anchoring the skin flap to Colles’ fascia anteriorly has lead to more consistent, reliable results following lifting. Other refinements such as direct undermining of the distal flap, performing adjunctive suction lipectomy, and using an anterior medial skin resection pattern have been associated with an improvement in overall results in those patients undergoing transverse thigh/buttock lifts but have not specifically decreased scar widening, traction deformities, and recurrent ptosis. Direct undermining, 3 to 4 cm beyond the planned line of resection, releases the superficial fascial attachments distally and allows for a greater lift. Suction lipectomy addresses fatty contour deformities that do not lie within the planned resection areas of the lift. The anterior medial skin resection pattern addresses skin laxity at the junction of the anterior and medial thigh and eliminates the need for incisions within the posterior buttock folds.

111
Q

A 43-year-old woman is unable to depress the left side of her lower lip after undergoing submental suction lipectomy. On follow-up examination three months later, she has persistent weakness of the lower lip. Which of the following is the most appropriate next step in management?

(A) Reassurance and continued observation
(B) Surgical exploration and nerve repair
(C) Injection of botulinum toxin into the unaffected side
(D) Nerve grafting
(E) Innervated free muscle transfer for facial reanimation

A

The correct response is Option A.

In this patient who has persistent weakness of the lower lip following submental suction lipectomy, the most appropriate next step is reassurance of the patient and continued observation. Submental suction lipectomy is considered to be a safe procedure as long as the cannula is passed superficial to the platysma; however, if the cannula is placed beneath the platysma, injury to the marginal mandibular branch of the facial nerve may result. According to one study, 81% of dissections found the marginal mandibular branch to be positioned above the inferior border of the mandible, while in 19% of dissections the nerve was positioned 1 cm below the mandible. Large studies of suction lipectomy patients have reported a rate of nerve injury of less than 1%. Because nerves and blood vessels are typically not transected during suction lipectomy, any resulting injuries are likely to be neurapraxias, which in most patients will completely resolve within three months.

Surgical exploration is unnecessary in a patient who has only a slight risk for nerve transection. Injection of botulinum toxin may result in facial symmetry but may also worsen symptoms or make common tasks (such as applying lipstick) more difficult. Nerve grafting and muscle transfers for facial reanimation are not indicated because the deficit will most likely resolve spontaneously. These procedures are typically reserved for correction of a significant facial nerve defect.

112
Q

Which of the following is the most common cause of death following suction lipectomy?

(A) Abdominal perforation
(B) Anesthetic complications
(C) Fat embolism
(D) Infection
(E) Thromboembolism

A

The correct response is Option E.

The incidence of fatalities associated with suction lipectomy performed in the outpatient setting is one in every 5000 procedures. According to a recent study of deaths associated with suction lipectomy procedures, in those patients in whom a cause of death was definitively established, 23% of the fatalities were shown to have resulted from thromboembolism. In contrast, 15% of fatalities resulted from abdominal wall perforation (with or without organ perforation), 10% involved anesthetic complications, 8% involved fat embolism, and only 5% were due to infection. Because lidocaine screening is rarely performed, any potential link between lidocaine toxicity and the development of the complications listed above was undetermined. Many of the reported deaths occurred during the first 24 hours following patient discharge. Other risk factors associated with suction lipectomy include aspiration of large amounts of tissue, increased volume of tumescent injection, and concomitantly performed procedures.

113
Q

In patients undergoing brachioplasty, which of the following is the most common long-term unfavorable result?

(A) Intermittent sharp pain in the arm
(B) Lymphedema of the hand and forearm
(C) Numbness of the medial arm
(D) Seroma of the upper arm
(E) Widening of the scar

A

The correct response is Option E.

Widened scars are the most common long-term complication following brachioplasty. These scars, which are typically located on the posteromedial upper arms, are red and visible for a minimum of one year and in fact may never completely fade. Patients should be informed of the potential for widened, visible scars prior to undergoing the procedure.

Intermittent sharp pain, lymphedema, numbness, and seromas can be complications of brachioplasty but are most likely to be temporary and to resolve within one to four weeks.

114
Q

Which of the following nerves is NOT at risk for injury during abdominoplasty?

(A) Genitofemoral
(B) Iliohypogastric
(C) Ilioinguinal
(D) Intercostal

A

The correct response is Option A.

During abdominoplasty, there is an increased risk for nerve entrapment or injury to the iliohypogastric and ilioinguinal nerves because of their anatomic location. Although the intercostal nerves are less prone to injury, they still lie in the region of the abdominoplasty. Because of the potential for injury, patients who have localized pain, paresthesias, and/or tenderness in the distribution of any of these nerves should undergo complete evaluation.

The genitofemoral nerve originates from L1-2 and courses deep in the abdominal wall. It pierces the fascia below the inguinal ligament and supplies sensation to the skin of the femoral triangle and pubis. Because this nerve lies inferior and deep to the abdominoplasty incision, it is not at risk for injury during an abdominoplasty procedure.

115
Q

For each patient, select the most appropriate management (A-E).

(A) Suction lipectomy
(B) Mini-abdominoplasty
(C) Full abdominoplasty
(D) Lower body lift
(E) Panniculectomy

1) A 35-year-old woman who has lost 150 lb following a gastric bypass procedure
2) A 45-year-old woman who weighs 400 lb and has a large, overhanging area of skin in the lower abdomen with ulceration

A

The correct response for Item 1 is Option D and for Item 2 is Option E.

In each of these patients who desires improved abdominal contour, the optimal procedure can be determined by evaluating the patient’s skin tone, abdominal wall musculature, and fat distribution.

The 35-year-old woman who had a massive reduction in weight following gastric bypass should undergo a lower body lift procedure. This will remove the excess skin and fat in the lower abdomen and thighs typically seen in patients who have lost an extensive amount of weight.

The obese 45-year-old woman who has a large, overhanging area of skin and fat (pannus) with ulceration should undergo panniculectomy. In this patient, gastric bypass may be performed either simultaneously or prior to the panniculectomy procedure.
Suction lipectomy alone is most appropriate for correction of localized abdominal protuberance in patients who have good skin tone and firm musculature in the abdominal wall. Mini-abdominoplasty is useful for removal of mild amounts of lower abdominal skin and fat. The length of the scar is limited with this procedure and the umbilicus is not altered. Full abdominoplasty removes the excess skin and fat in the lower abdomen but relocates the umbilicus and results in an elongated scar. This procedure does not correct excess skin in the flanks and upper thighs.

116
Q

Following abdominoplasty, skin necrosis is most likely to occur at which of the following sites?

(A) Epigastrium
(B) Flanks
(C) Periumbilical region
(D) Suprapubic region
(E) Umbilicus

A

The correct response is Option D.

Following abdominoplasty, skin necrosis is most likely to occur in the suprapubic region. The blood supply to the abdominoplasty is derived from the lateral interstitial vessels; because of this, the lower midline region of the flap is most at risk for devascularization. In addition, downward transposition of any pre-existing scars on the upper abdomen will limit blood flow and ultimately inhibit flap vascularization.

Because the epigastrium and flanks are in close proximity to the base of the elevated skin flap, they are better protected and at less risk for development of necrosis. Although umbilical necrosis is rare, it can occur as a result of excessive defatting of the umbilical stalk, tension during attachment to the abdominal skin, or compression of the stalk during plication of the rectus.

117
Q

A 43-year-old woman who recently lost 45.5 kg (100 lb) has severe skin laxity of the arms with moderate fat deposition. The most likely cause of her current findings is loosening of which of the following fascia?

(A) Clavipectoral
(B) Colles’
(C) Deltoid
(D) Pectoralis major
(E) Scarpa’s

A

The correct response is Option A.

This patient’s skin laxity is most likely caused by a loosening of the clavipectoral fascia. Anatomic studies have shown that in youth the soft tissues of the posteromedial arm are firmly suspended to a tough yet dynamic fascial system sling that ultimately gains its strength from the clavicular periosteum by means of the clavipectoral and axillary fasciae. The clavipectoral fascia lies deep to the pectoralis major muscle and extends from the clavicle to the dome of the axillary fascia. Loosening of these connections, combined with relaxation of the fascia itself with age, weight fluctuations, and gravitational pull, results in significant ptosis of the posteromedial arm. Other mechanisms contributing to arm ptosis include relaxation and stretching of the skin and superficial fascial system of the arm, as well as flaccidity of the posterior arm muscles resulting from age and lack of exercise. Fascial anchoring brachioplasty is used to correct this deformity.

Fascial anchoring and suspension of the superficial fascial system can be used in body contouring of other sites in the trunk and extremities, including Colles’ fascia in the medial thigh lift and Scarpa’s fascia in abdominoplasty. The deltoid and pectoralis major fascia have no effect on upper arm laxity.

118
Q

Which of the following is the most common complication of performing full abdominoplasty in combination with suction lipectomy?

(A) Infection
(B) Nerve injury
(C) Seroma formation
(D) Skin necrosis
(E) Wound dehiscence

A

The correct response is Option C.

The most common complication of a full abdominoplasty performed in conjunction with suction lipectomy is the formation of seromas. Although this remains a serious problem, steps that can be taken to reduce the potential risk for the development of seromas include avoiding electrocoagulation for dissection, limiting the quantity of local anesthetic used, securing the flap with quilting sutures, and maintaining adequate wound drainage.

Infection, nerve injury, skin necrosis, and wound dehiscence are all less common complications. The prophylactic use of antibiotics and antibiotic irrigation will help to minimize the potential for wound infection. The risk for skin necrosis and skin slough is increased when abdominoplasty procedures are combined with suction lipectomy; a history of smoking or diabetes mellitus, as well as the presence of abdominal scars, can also increase the patient’s risk for necrosis. Wound dehiscence can result from inferior surgical techniques, excess wound tension, and insufficient placement of deep dermal sutures.

119
Q

A patient develops a supraumbilical bulge after undergoing full abdominoplasty and suction lipectomy of the abdomen and flanks. Which of the following is the most likely cause?

(A) Failure to plicate the rectus muscle
(B) Inadequate skin resection
(C) Inadequate suction lipectomy
(D) Loose fascial suspension
(E) Umbilical malpositioning

A

The correct response is Option A.

In this patient who has developed a supraumbilical bulge after undergoing a combined full abdominoplasty and suction lipectomy procedure, the most likely cause is a failure to plicate the rectus muscle. In standard abdominoplasty procedures, a low horizontal or W-shaped skin incision is made. The rectus diastasis is typically marked and then directly plicated both above and below the umbilicus and tapered superiorly toward the xiphoid. Alternately, the surgeon can create an incision into the fascia and then plicate the muscle in the same manner. If neither of these techniques is performed, a relative laxity of the upper abdominal region can develop, resulting in the appearance of a supraumbilical bulge.

Fixation of the skin flap to the umbilicus will relieve tension in the lower portion of the flap. Telescoping of the umbilicus is performed in the midline at the level of the anterosuperior iliac spine. Suction lipectomy is typically used for reduction of redundant fat deposits along the flanks, waistline, and iliac regions. The suctioning should be adjusted on a patient-by-patient basis. Repair of the superficial fascial system will diffuse tension on the skin flap, lift areas of soft-tissue excess more effectively, and provide lasting support. This technique is now considered critical to the success of body contouring procedures involving the trunk and extremities.

120
Q

A 45-year-old man desires cosmetic improvement of skin ptosis of the posteromedial arms. He has lost 50 lb over the past year, but is currently 20 lb above his ideal body weight. Which of the following is the most appropriate management?

(A) Continued weight loss
(B) Exercise program
(C) Brachioplasty
(D) Suction-assisted lipectomy
(E) Ultrasound-assisted lipectomy

A

The correct response is Option C.

The most appropriate next step in the management of this patient with arm ptosis is brachioplasty. Brachioplasty is most effective in patients with extensive skin ptosis of the arm. This condition is caused by loosening of the connections of the superficial fascial system of the arm to the axillary fascia. It may occur in conjunction with the aging process or with gravitational pull and/or fluctuations in body weight. During the surgical procedure, the arm flap is securely anchored to the axillary fascia. Complications following this procedure are minimal.

Instead of helping to resolve the ptosis, further weight loss can actually worsen it. In the same way, suction-assisted lipectomy can worsen the ptosis by creating further laxity in the overlying skin. A regimented exercise program is only effective for patients with minimal skin and soft-tissue laxity. The thermal effect of ultrasound-assisted lipectomy will tighten the skin but will not correct the underlying ptosis.